Download as docx, pdf, or txt
Download as docx, pdf, or txt
You are on page 1of 90

E.

Essential elements
Gulf Resorts v. Philippine Charter Ins., GR 156167, 16 May 2005 - YAP

GULF RESORTS, INC., Petitioner, v. PHILIPPINE CHARTER INSURANCE CORPORATION,


Respondent.
 
DOCTRINE:

Section 2(1) of the Insurance Code defines a contract of insurance as an agreement whereby one
undertakes for a consideration to indemnify another against loss, damage or liability arising from an
unknown or contingent event. Thus, an insurance contract exists where the following elements concur:

1. The insured has an insurable interest;

2. The insured is subject to a risk of loss by the happening of the designated peril;

3. The insurer assumes the risk;

4. Such assumption of risk is part of a general scheme to distribute actual losses among a large group of
persons bearing a similar risk; andcralawlibrary

5. In consideration of the insurer's promise, the insured pays a premium. (Emphasis ours) 26

 
FACTS:

[P]laintiff is the owner of the Plaza Resort situated at Agoo, La Union and had its properties in said resort
insured originally with the American Home Assurance Company (AHAC-AIU).
 
After the earthquake, petitioner advised respondent that it would be making a claim under its Insurance
Policy No. 31944 for damages on its properties. Respondent instructed petitioner to file a formal claim,
then assigned the investigation of the claim to an independent claims adjuster, Bayne Adjusters and
Surveyors, Inc. 3

 On July 30, 1990, respondent, through its adjuster, requested the petitioner to submit various documents
in support of its claim. 

On August 7, 1990, Bayne Adjusters and Surveyors, Inc., through its Vice-President A.R. de Leon, 4

rendered a preliminary report finding extensive damage caused by the earthquake to the clubhouse and
5

to the two swimming pools. 

Mr. de Leon stated that "except for the swimming pools, all affected items have no coverage for
earthquake shocks." On August 11, 1990, petitioner filed its formal demand for settlement of the damage
6

to all its properties in the Agoo Plaza Resort. On August 23, 1990, respondent denied petitioner's claim
on the ground that its insurance policy only afforded earthquake shock coverage to the two swimming
pools of the resort. Petitioner and respondent failed to arrive at a settlement.
8 9

On February 21, 1994, the lower court after trial ruled in favor of the respondent, viz:

 plaintiff paid only a premium of P393.00 against the peril of earthquake shock, the same premium it paid against earthquake
shock only on the two swimming pools in all the policies issued by AHAC(AIU) (Exhibits "C", "D", "E", "F" and "G"). From this fact
the Court must consequently agree with the position of defendant that the endorsement rider (Exhibit "7-C") means that only the
two swimming pools were insured against earthquake shock.
Plaintiff correctly points out that a policy of insurance is a contract of adhesion hence, where the language used in an insurance
contract or application is such as to create ambiguity the same should be resolved against the party responsible therefor, i.e., the
insurance company which prepared the contract. To the mind of [the] Court, the language used in the policy in litigation is clear
and unambiguous hence there is no need for interpretation or construction but only application of the provisions therein.

From the above observations the Court finds that only the two (2) swimming pools had earthquake shock coverage and were
heavily damaged by the earthquake which struck on July 16, 1990. Defendant having admitted that the damage to the swimming
pools was appraised by defendant's adjuster at P386,000.00, defendant must, by virtue of the contract of insurance, pay plaintiff
said amount.

 
ISSUE:
 WHETHER THE COURT OF APPEALS CORRECTLY HELD THAT UNDER RESPONDENT'S
INSURANCE POLICY NO. 31944, ONLY THE TWO (2) SWIMMING POOLS, RATHER THAN ALL THE
PROPERTIES COVERED THEREUNDER, ARE INSURED AGAINST THE RISK OF EARTHQUAKE
SHOCK.
 
 
RULING:

Yes. The RTC and Court of Appeals are both correct.

Petitioner contends that pursuant to this rider, no qualifications were placed on the scope of the
earthquake shock coverage. Thus, the policy extended earthquake shock coverage to all of the insured
properties.

It is basic that all the provisions of the insurance policy should be examined and interpreted in
consonance with each other. All its parts are reflective of the true intent of the parties. The policy cannot
25

be construed piecemeal. Certain stipulations cannot be segregated and then made to control; neither do
particular words or phrases necessarily determine its character. Petitioner cannot focus on the
earthquake shock endorsement to the exclusion of the other provisions. All the provisions and riders,
taken and interpreted together, indubitably show the intention of the parties to extend earthquake shock
coverage to the two swimming pools only.

A careful examination of the premium recapitulation will show that it is the clear intent of the parties to
extend earthquake shock coverage only to the two swimming pools. 

Section 2(1) of the Insurance Code defines a contract of insurance as an agreement whereby one
undertakes for a consideration to indemnify another against loss, damage or liability arising from an
unknown or contingent event. Thus, an insurance contract exists where the following elements concur:

1. The insured has an insurable interest;

2. The insured is subject to a risk of loss by the happening of the designated peril;

3. The insurer assumes the risk;

4. Such assumption of risk is part of a general scheme to distribute actual losses among a large group of
persons bearing a similar risk; and

5. In consideration of the insurer's promise, the insured pays a premium.


An insurance premium is the consideration paid an insurer for undertaking to indemnify the insured
against a specified peril. In fire, casualty, and marine insurance, the premium payable becomes a debt
27

as soon as the risk attaches.  


28

In the subject policy, no premium payments were made with regard to earthquake shock coverage,
except on the two swimming pools. There is no mention of any premium payable for the other resort
properties with regard to earthquake shock.

Philamcare Health Systems v. CA, GR 125678, 18 March 2002

Facts:

Ernani Trinos, deceased husband of private respondent Julita Trinos, was approved for a health care
coverage with petitioner from March 1988 to March1989. The same was extended twice until June 1990.
During the period of his coverage, Ernani was hospitalized several times, however, petitioner denied the
claim of private respondent because the Health Care Agreement was allegedly void due to the alleged
concealment of Ernani that he was not hypertensive, diabetic, and asthmatic, contrary to his answer in the
application form.

Petitioner argues that the agreement merely granted living benefits, such as check-ups and
hospitalisation, hence it is not an insurance contract. Petitioner further argues that it is not an insurance
company, which is governed by the Insurance Commission, but a Health Maintenance Organization under
the authority of the Department of Health.

Issues:

1. Whether or not the Health Care Agreement between the deceased and the petitioner falls under
the ambit of an insurance contract.
2. Whether the alleged concealment of the deceased will invalidate the Agreement.

Ruling:

1. Yes. In the case at bar, the insurable interest of respondents husband in obtaining the health care
agreement was his own health. Section 10 of the Insurance Code is clear that every person
has an insurable interest in the life and health of himself.  The health care agreement was
in the nature of non-life insurance, which is primarily a contract of indemnity. Once the
member incurs hospital, medical or any other expense arising from sickness, injury or
other stipulated contingent, the health care provider must pay for the same to the extent
agreed upon under the contract

2. No. The answer assailed by petitioner was in response to the question relating to the medical
history of the applicant. This largely depends on opinion rather than fact, especially coming from
respondents husband who was not a medical doctor. Where matters of opinion or judgment are
called for, answers made in good faith and without intent to deceive will not avoid a policy even
though they are untrue. (A)lthough false, a representation of the expectation, intention, belief,
opinion, or judgment of the insured will not avoid the policy if there is no actual fraud in inducing
the acceptance of the risk, or its acceptance at a lower rate of premium, and this is likewise the
rule although the statement is material to the risk, if the statement is obviously of the foregoing
character, since in such case the insurer is not justified in relying upon such statement, but is
obligated to make further inquiry. There is a clear distinction between such a case and one in
which the insured is fraudulently and intentionally states to be true, as a matter of expectation or
belief, that which he then knows, to be actually untrue, or the impossibility of which is shown by
the facts within his knowledge, since in such case the intent to deceive the insurer is obvious and
amounts to actual fraud. Under Section 27 of the Insurance Code, a concealment entitles the
injured party to rescind a contract of insurance. The right to rescind should be exercised previous
to the commencement of an action on the contract.

F. Applicability of Civil Code

Insular Life v. Ebrado, GR 44059, 28 October 1977

Facts:

Buenaventura was married to Pascuala with whom he had six. Later on however, he started living with
Carponia although he was still legally married to Pascuala and had not legally separated from her.

While living with Carponia, Buenaventura obtained an insurance policy from Insular Life Assurance Co.
with a rider for accidental death benefit and designated Carponia as the revocable beneficiary, referring
her therein as his wife. Barely more than a year after obtaining the policy, Buenaventura died when he
was hit by a falling branch of a tree.

Carponia filed a claim for the proceeds of the Policy as the designated beneficiary therein, although she
admits that she and the insured Buenaventura were merely living as husband and wife without the benefit
of marriage. Pascuala also filed her claim as the widow of the deceased insured. She asserts that she is
the one entitled to the insurance proceeds, not the common-law wife, Carponia. In view of the conflicting
claims, Insular Life brought the matter to court interpleading both parties in the case. The trial court ruled
in favor of Pascuala.

Issue:
Can a common-law wife named as beneficiary in the life insurance policy of a legally married man claim
the proceeds thereof in case of death of the latter?

Held:

No. In essence, a life insurance policy is no different from a civil donation insofar as the
beneficiary is concerned. Both are founded upon the same consideration: liberality. A beneficiary is like
a donee, because from the premiums of the policy which the insured pays out of liberality, the beneficiary
will receive the proceeds or profits of said insurance. As a consequence, the proscription in Article 739 of
the new Civil Code should equally operate in life insurance contracts. The mandate of Article 2012
cannot be laid aside: any person who cannot receive a donation cannot be named as beneficiary in
the life insurance policy of the person who cannot make the donation.

Policy considerations and dictates of morality rightly justify the institution of a barrier between
common law spouses in regard to property relations since such relationship ultimately encroaches
upon the nuptial and filial rights of the legitimate family. 

A conviction for adultery or concubinage is not necessary. Article 739 itself provides that the guilt of
the donee may be proved by preponderance of evidence in the same action. In this case, the
common law relationship is already admitted by Carponia herself in the stipulation of facts they
submitted to the court.
Zenith Insurance v. CA, GR 85296, 14 May 1990- YAP

ZENITH INSURANCE CORPORATION, petitioner,


vs.
COURT OF APPEALS and LAWRENCE FERNANDEZ, respondents.
 
DOCTRINE:
The award of damages in case of unreasonable delay in the payment of insurance claims is governed by
the Philippine Insurance Code.  As regards the award of moral and exemplary damages, the rules
under the Civil Code of the Philippines shall govern.
FACTS:
 
On January 25, 1983, private respondent Lawrence Fernandez insured his car for "own
damage" under private car Policy No. 50459 with petitioner Zenith Insurance Corporation. On July 6,
1983, the car figured in an accident and suffered actual damages in the amount of P3,640.00. After
allegedly being given a run around by Zenith for two (2) months, Fernandez filed a complaint with the
Regional Trial Court of Cebu for sum of money and damages resulting from the refusal of Zenith to pay
the amount claimed. The complaint was docketed as Civil Case No. CEB-1215. Aside from actual
damages and interests, Fernandez also prayed for moral damages in the amount of P10,000.00,
exemplary damages of P5,000.00, attorney's fees of P3,000.00 and litigation expenses of P3,000.00.
On September 28, 1983, Zenith filed an answer alleging that it offered to pay the claim of Fernandez
pursuant to the terms and conditions of the contract which, the private respondent rejected.
 
RULING OF THE TRIAL COURT:
a decision was rendered by the trial court in favor of private respondent Fernandez.

RULING OF THE COURT OF APPEALS:


On June 10, 1986, petitioner filed a notice of appeal before the trial court. The notice of appeal was
granted in the same order granting private respondent's motion for execution pending appeal. The appeal
to respondent court assigned the following errors:

I. The lower court erred in denying defendant appellant to adduce evidence in its behalf.
II. The lower court erred in ordering Zenith Insurance Corporation to pay the amount of
P3,640.00 in its decision.
III. The lower court erred in awarding moral damages, attorneys fees and exemplary
damages, the worst is that, the court awarded damages more than what are prayed for in
the complaint. (p. 12, Rollo)

On August 17, 1988, the Court of Appeals rendered its decision affirming in toto the decision of the
trial court. It also ruled that the matter of the trial court's denial of Fernandez's right to adduce
evidence is a closed matter in view of its (CA) ruling in AC-G.R. 04644 wherein Zenith's petition
questioning the trial court's order submitting the case for decision without Zenith's evidence, was
dismissed.

Petitioner contends that while the complaint of private respondent prayed for P10,000.00 moral damages,
the lower court awarded twice the amount, or P20,000.00 without factual or legal basis; while private
respondent prayed for P5,000.00 exemplary damages, the trial court awarded P20,000.00; and while
private respondent prayed for P3,000.00 attorney's fees, the trial court awarded P5,000.00.

ISSUE:
What is the basis of the court in awarding the damages to Hernandez?

RULING:
The award of damages in case of unreasonable delay in the payment of insurance claims is governed by
the Philippine Insurance Code, which provides:

Sec. 244. In case of any litigation for the enforcement of any policy or contract of
insurance, it shall be the duty of the Commissioner or the Court, as the case may be, to
make a finding as to whether the payment of the claim of the insured has been
unreasonably denied or withheld; and in the affirmative case, the insurance company
shall be adjudged to pay damages which shall consist of attorney's fees and other
expenses incurred by the insured person by reason of such unreasonable denial
or withholding of payment plus interest of twice the ceiling prescribed by the
Monetary Board of the amount of the claim due the insured, from the date following
the time prescribed in section two hundred forty-two or in section two hundred forty-three,
as the case may be, until the claim is fully satisfied; Provided, That the failure to pay any
such claim within the time prescribed in said sections shall be considered prima facie
evidence of unreasonable delay in payment.

It is clear that under the Insurance Code, in case of unreasonable delay in the payment of the proceeds of
an insurance policy, the damages that may be awarded are: 

1) attorney's fees; 
2) other expenses incurred by the insured person by reason of such unreasonable denial or withholding of
payment; 
3) interest at twice the ceiling prescribed by the Monetary Board of the amount of the claim due the
injured; and
 4) the amount of the claim.

As regards the award of moral and exemplary damages, the rules under the Civil Code of the
Philippines shall govern.

"The purpose of moral damages is essentially indemnity or reparation, not punishment or correction.
Moral damages are emphatically not intended to enrich a complainant at the expense of a defendant, they
are awarded only to enable the injured party to obtain means, diversions or amusements that will serve to
alleviate the moral suffering he has undergone by reason of the defendant's culpable action." (J. Cezar S.
Sangco, Philippine Law on Torts and Damages, Revised Edition, p. 539) (See also R and B Surety &
Insurance Co., Inc. v. IAC, G.R. No. 64515, June 22, 1984; 129 SCRA 745). While it is true that no proof
of pecuniary loss is necessary in order that moral damages may be adjudicated, the assessment of which
is left to the discretion of the court according to the circumstances of each case (Art. 2216, New Civil
Code), it is equally true that in awarding moral damages in case of breach of contract, there must be a
showing that the breach was wanton and deliberately injurious or the one responsible acted fraudulently
or in bad faith (Perez v. Court of Appeals, G.R. No. L-20238, January 30,1965; 13 SCRA 137; Solis v.
Salvador, G.R. No. L-17022, August 14, 1965; 14 SCRA 887).

In the instant case, there was a finding that private respondent was given a "run-around" for two
months, which is the basis for the award of the damages granted under the Insurance Code for
unreasonable delay in the payment of the claim.

However, the act of petitioner of delaying payment for two months cannot be considered as
wanton or malevolent to justify an award of P20,000.00 as moral damages, taking into
consideration also the fact that the actual damage on the car was only P3,460.

In the pre-trial of the case, it was shown that there was no total disclaimer by respondent. The
reason for petitioner's failure to indemnify private respondent within the two-month period was
that the parties could not come to an agreement as regards the amount of the actual damage on
the car. The amount of P10,000.00 prayed for by the private respondent as moral damages is
equitable.
On the other hand, exemplary or corrective damages are imposed by way of example or correction for the
public good (Art. 2229, New Civil Code of the Philippines). In the case of Noda v. Cruz-Arnaldo, G.R. No.
57322, June 22,1987; 151 SCRA 227, exemplary damages were not awarded as the insurance company
had not acted in wanton, oppressive or malevolent manner. The same is true in the case at bar.
 

G. SUBROGATION

Fireman's Fund et al v Jamila et al., GR 27427, 7 April 1976 - BONDOC


 
CASE TITLE:
FIREMAN’S FUND INSURANCE COMPANY and FIRESTONE TIRE and RUBBER COMPANY OF THE
PHILIPPINES  v. JAMILA & COMPANY, INC. and FIRST QUEZON CITY INSURANCE CO., INC.
G.R. No. 27427
Date:  April 7, 1976.
Ponente: J. AQUINO.
 
 
FACTS:
Firestone TIRE and RUBBER COMPANY
--> suffered losses of properties valued at P11,925
--> allegedly due to the acts of its employees
--> who connived with security guards from the agency Jamila & Co.
      --> provided under contract by Jamila & Co., Inc.
 
INSURANCE CLAIM:
The amount of loss was recovered from its insurer,
the Fireman’s Fund Insurance Co.
 
 
SUBROGATION:
Fireman’s Fund Insurance Co., together with the insured,
--> sued Jamila (security agency) and
--> its Surety, (the First Quezon City Insurance Co., Inc.)
--> to recover the amount of loss
--> on the basis of legal subrogation.
 
RTC DISMISSED IT (!)
 
(1) as to Jamila (the agency), the RTC dismissed the complaint
--> on the ground that the plaintiff had no cause of action against it
FOR LACK OF THE DEBTOR’S CONSENT to the subrogation; and
 
(2) as to First Quezon City Insurance Co., Inc.,
the RTC dismissed the complaint
--> on the ground of res judicata.
--> apparently, the same action was previously filed (in Civil Case No. 56311)
      --> which was dismissed because of the failure to appear at the pre-trial  of the same plaintiffs
and their counsel
 
Note: THE RTC ISSUED “SEESAW” RULINGS.
(see case if asked)
 
ISSUE:
W/N the Right to LEGAL SUBROGATION requires CONSENT of the DEBTOR,
as claimed by JAMILA and their Surety.
 
HELD:
No, consent of debtor is not required.
 
The Supreme Court held that;
(1) that the action is sanctioned by Article 2207 of the Civil Code;
(2) that Fireman’s Fund, as the insurer, is entitled to go after the person or entity that violated its
contractual commitment to answer for the loss insured against;
(3) sufficient ultimate facts were alleged in the complaint to sustain the cause of action against Jamila.
Thus, hindi dapat na-dismiss.
 
RATIO:
(1) RIGHT of SUBROGATION IS NOT DEPENDENT UPON CONTRACT OR ASSIGNMENT OF CLAIM
--> When the insurance company pays for the loss, such payment operates as an equitable
assignment to the insurer of the property and all remedies which the insured may have for the
recovery thereof.

That right is not dependent upon, nor does it grow out of, any privity of contract, or upon written
assignment of claim, and payment to the insured makes the insurer an assignee in equity.
 
(2) RIGHT OF SUBROGATION UNDER ART. 2207 OF THE CIVIL CODE; ORIGIN AND NATURE —
Article 2207 is a restatement of a settled principle of American jurisprudence. Subrogation has been
referred to as the doctrine of substitution. It is an arm of equity that may guide or even force one to pay a
debt for which an obligation was incurred but which was in whole or in part paid by another. Subrogation
is founded on principles of justice and equity, and its operation is governed by principles of equity. It rests
on the principle that substantial justice should be attained regardless of form, that is, its basis is the doing
of complete, essential, and perfect justice between all the parties without regard to form.
 
 

Pan Malayan Insurance v CA, GR 81026, 3 April 1990-


FACTS:

The Petitioner, PANMALAY filed a complaint for damages with the RTC of Makati
against private respondents Erlinda Fabie and her driver. PANMALAY averred that:
- it insured a Mitsubishi Colt Lancer car with plate No. DDZ-431 and registered in
the name of Canlubang Automotive Resources Corporation.
-On May 26, 1985, due to the "carelessness, recklessness, and imprudence" of
the unknown driver of a pick-up with plate no. PCR-220, the insured car was hit and
suffered damages.
- PANMALAY defrayed the cost of repair of the insured car and, therefore,
was subrogated to the rights of CANLUBANG against the driver of the pick-up and
his employer, Erlinda Fabie;
-despite repeated demands, defendants, failed and refused to pay the claim of
PANMALAY.
Private respondents, thereafter, filed a Motion for Bill of Particulars and a supplemental
motion thereto. In compliance therewith, PANMALAY clarified, among others, that the
damage caused to the insured car was settled under the "own damage", coverage of
the insurance policy, and that the driver of the insured car was, at the time of the
accident, an authorized driver duly licensed to drive the vehicle. PANMALAY also
submitted a copy of the insurance policy and the Release of Claim and Subrogation
Receipt executed by CANLUBANG in favor of PANMALAY.
On February 12, 1986, respondents filed a Motion to Dismiss alleging that PANMALAY
had no cause of action against them. And argued that payment under the "own
damage" clause of the insurance policy precluded subrogation under Article 2207 of the
Civil Code, since indemnification thereunder was made on the assumption that there
was no wrongdoer or no third party at fault.
RTC : dismissing PANMALAY's complaint for no cause of action and denied
PANMALAY's motion for reconsideration.
CA: same as RTC decision.

ISSUE: whether or not the insurer PANMALAY may institute an action to recover the
amount it had paid its assured in settlement of an insurance claim against private
respondents as the parties allegedly responsible for the damage caused to the insured
vehicle.

Ruling: Yes.
PANMALAY is correct.

Article 2207 of the Civil Code is founded on the well-settled principle of subrogation.

If the insured property is destroyed or damaged through the fault or negligence of a


party other than the assured, then the insurer, upon payment to the assured, will be
subrogated to the rights of the assured to recover from the wrongdoer to the
extent that the insurer has been obligated to pay.

Payment by the insurer to the assured operates as an equitable assignment to the


former of all remedies which the latter may have against the third party whose
negligence or wrongful act caused the loss. The right of subrogation is not dependent
upon, nor does it grow out of, any privity of contract or upon written assignment of claim.
It accrues simply upon payment of the insurance claim by the insurer

Exceptions to this rule:

-For instance, if the assured by his own act releases the wrongdoer or third
party liable for the loss or damage, from liability, the insurer's right of subrogation is
defeated.

-Similarly, where the insurer pays the assured the value of the lost goods
without notifying the carrier who has in good faith settled the assured's claim for
loss, the settlement is binding on both the
-where the insurer pays the assured for a loss which is not a risk covered
by the policy, thereby effecting "voluntary payment", the former has no right of
subrogation against the third party liable for the loss

[Lasheras] G.R. No. L-22146             September 5, 1967

SVERIGES ANGFARTYGS ASSURANS FORENING vs. QUA CHEE GAN

FACTS:

·         On August 23 and 24, 1947, defendant Qua Chee Gan, a sole proprietorship, shipped on
board the S.S. NAGARA, as per bills of lading Exhs. A and B, 2,032,000 kilos of bulk copra at
Siain, Quezon, consigned to DAL International Trading Co., in Gdynia, Poland.

·         The vessel first called at the port of Karlshamn, Sweden, where it unloaded 969,419 kilos of
bulk copra. Then, it proceeded to Gdynia where it unloaded the remaining copra shipment.

·         The actual outturn weights in the latter port showed that only 1,569,429 kilos were
discharged. Because of the alleged confirmed cargo shortage, the Polish cargo insurers had to
indemnify the consignee for the value thereof.

·         Thereafter, the former(insurer) sued the shipowner, the Swedish East Asia Company, in
Gothenburg, Sweden. The latter, in turn sued, defendant and had it summoned to Gothenburg.

·         Defendant, however, refused to submit to that court's jurisdiction and its objection was
sustained.

·         In March, 1951, a settlement was effected between the Polish cargo insurers and the
shipowner. Plaintiff, as the indemnity insurer for the latter, paid approximately $60,733.53 to the
Polish insurers.

·         On August 16, 1954, claiming to have been subogated to the rights of the carrier, plaintiff
sued defendant before the CFI of Manila to recover U.S. $60,733.53 plus 17% exchange tax,
with legal interest, as the value of the alleged cargo short shipment, and P10,000 as attorney's
fees.

·         On August 1, 1955, defendant filed a motion to dismiss on the ground of prescription under
the Carriage of Goods by Sea Act.

CFI: sustained the motion

SC: (Sveriges Angfartygs Assurans Forening v. Qua Chee Gan, L-9757, April 16, 1959.) : 
reversed the order of dismissal and remanded the case for further proceedings.
 

CFI: rendered its decision dismissing the complaint and awarding P10,000 as attorney's fees to
defendant. It ruled

(a) that there was no short shipment on defendant's part;

(b) that plaintiff's insurance policy did not cover the short shipment, and

(c) defendant was merely acting as an agent of Louis Dreyfus & Co., who was the real
shipper.

Plaintiff’s Contention:

Plaintiff theorizes that defendant had two shipments at Siain, Quezon province (1) 812,800 kilos
for Karlshamn and (2) 2,032,000 kilos for Gdynia. The Karlshamn shipment was asserted to
have been covered by a separate bill of lading which, however, was allegedly lost subsequently.

ISSUE:

Whether or not plaintiff cannot recover what it paid because the insurance policy did not cover
the short shipment, thus there is no subrogation

RULING:

The plaintiff cannot recover what it paid. No shortage.

NO SUBROGATION

General Rule:

The rule is that an insurer who pays the insured for loss or liability not covered by the
policy is not subrogated to the latter.

Exception:

However, even assuming that there was unwarranted — or "volunteer" — payment, plaintiff
could still recover what it paid — in effect — to the carrier from defendant shipper under
Art. 1236 of the Civil Code which allows a third person who pays on behalf of another to recover
from the latter, although there is no subrogation.
But since the payment here was without the knowledge and consent of defendant,
plaintiff's right of recovery is defeasible by the former's defenses since the Code is clear
that the recovery is only up to the amount by which the defendant was benefited.

NO SHORTAGE

Plaintiff's cause of action suffers from several fatal defects and inconsistencies. The alleged
shipment of 812,800 kilos for Karlshamn is contradicted by plaintiff's admission in paragraphs 2
and 3 of its complaint that defendant shipped only 2,032,000 kilos of copra at Siain, purportedly
for both Gdynia and Karlshamn.Needless to state, plaintiff is bound by such judicial admission.

Moreover, the alleged existence of the Karlshamn bills of lading is negatived by the fact that
Exhibits A and B — the bills of lading presented by plaintiff — show that the 2,032,000 kilos of
copra loaded in Siain were for Gdynia only. Further destroying its case is the testimony of
plaintiff's own witness, Mr. Claro Pasicolan, who on direct examination affirmed that these two
exhibits connstituted the complete set of documents which the shipping agent in charge of the
vessel S.S. NAGARA issued covering the copra cargo loaded at Siain.

In view of this admission and for want of evidentiary support, plaintiff's belated claim that there
is another complete set of documents cannot be seriously taken.

NO PREPONDERANT EVIDENCE

Now the fact that the sum total of the cargo unloaded at Karlshamn and Gdynia would exceed
what appears to have been loaded at Siain by as much as 233,848 kilos can only show that
defendant really overshipped, not shortshipped. And while this would not tally with defendant's
claim of having weighed the copra cargo 100% at Siain, thus exposing a flaw in defendant's
case, yet it is elementary that plaintiff must rely on the strength of its own case to recover, and
not bank on the weakness of the defense. Plaintiff here failed to establish its case by
preponderance on evidence.

DEFENDANT IS AN AGENT

On the question whether defendant is the real shipper or merely an agent of Louis Dreyfus &
Co., suffice it to say that altho on Exhibits A and B his name appears as the shipper, yet the
very loading certificate, Exhibit 3 [5-Deposition of Horle], issued and signed by the Chief Mate
and Master of the S.S. NAGARA shows that defendant was acting merely for account of Louis
Dreyfus & Co. The other documentary exhibits confirm this. Anyway, in whatever capacity
defendant is considered, it cannot be liable since no shortshipment was shown.

Rizal Surety v Manila Railroad, GR L-24043, 25 April 1968 - CANADA

FACTS:

On Nov 29, 1960, a vessel named SS Flying Trader, loaded on board a cargo
which is an offset press machine, from Italy to Manila. Upon reaching the port of
destination and upon unloading it, it was dropped by the crane which resulted in
damages to the machine. The plaintiff as the insurer had paid the consignee,
Suter, Inc. the amount of P16,500.00 for the machine and P180.70 for the
International Adjustment Bureau as adjuster’s fee. However, under the
management contract the arrastre charges in this particular shipment was paid
on the weight or measurement basis whichever is higher, and not on the value
thereof.

ISSUE:

Can the insurance get an amount greater than what was declared?

HELD:

Plaintiff Insurance Company cannot recover from defendants an amount greater


than that to which the consignee could lawfully lay claim. The management
contract is clear, the amount is limited to P500.

 
If the plaintiff's property has been insured, and he has received indemnity from
the insurance company for the injury or loss arising out of the wrong or breach of
contract complained of, the insurance company shall be subrogated to the right
of the insured against the wrong-doer or the person who has violated the
contract. If the amount paid by the insurance company does not fully cover
the injury or loss, the aggrieved party shall be entitled to recover the
deficiency from the person causing the loss or injury.
 
The insurance company has no greater right than the party in interest
thereof.

St. Paul Fire v Macondray, GR L-27796, 25 March 1976- ACUÑA

FACTS:

On June 29, 1960, Winthrop Products, Inc., of New York, New York, U.S.A., shipped aboard
the SS "Tai Ping", owned and operated by Wilhelm Wilhelmsen 218 cartons and drums of
drugs and medicine, with the freight prepaid, which were consigned to Winthrop-Stearns Inc.,
Manila, Philippines (Consignee/shipper). Barber Steamship Lines, Inc (agent), agent of
Wilhelm Wilhelmsen issued Bill of Lading No. 34, in the name of Winthrop Products, Inc. as
shipper, with arrival notice in Manila to consignee Winthrop-Stearns, Inc., Manila, Philippines.
The shipment was insured by the shipper against loss and/or damage with the St. Paul Fire &
Marine Insurance Company (insurer) under its insurance Special Policy No. OC-173766 dated
June 23, 1960.

On August 7, 1960, the SS "Tai Ping" arrived at the Port of Manila and discharged its
aforesaid shipment into the custody of Manila Port Service. 

The said shipment was discharged complete and in good order with the exception of one (1)
drum and several cartons which were in bad order condition. 
Because the consignee failed to receive the whole shipment and as several cartons of
medicine were received in bad order condition, they filed the corresponding claim in the
amount of Fl,109.67 representing the Cost, Insurance and Freight (C.I.F.) value of the
damaged drum and cartons of medicine with the carrier, herein defendants- appellees and
the Manila Port Service. However, both refused to pay such a claim. consequently,

The consignee filed its claim with the insurer, St. Paul Fire & Marine insurance Co., and the
insurance company, on the basis of such claim, paid to the consignee the insured value of
the lost and damaged goods, including other expenses in connection therewith, in the total
amount of $1,134.46 U.S. currency.

As subrogee of the rights of the shipper and/or consignee, the insurer, St. Paul Fire & Marine
Insurance Co., instituted with the Court of First Instance of Manila an action against the
defendants for the recovery of said amount of $1,134.46, plus costs.

As their defense, Manila Port Service and Manila Railroad Company resisted the action,
contending, among others, that the whole cargo was delivered to the consignee in the same
condition in which it was received from the carrying vessel;

that their rights, duties and obligations are governed by and subject to the terms, conditions
and limitations contained in the Management Contract between the Bureau of Customs and
Manila Port Service,

and their liability is limited to the invoice value of the goods, but in no case more than
P500.00 per package, pursuant to paragraph 15 of the said Management Contract; 

and that they are not the agents of the carrying vessel in the receipt and delivery of cargoes
in the Port of Manila.

Macondray & Co., Inc., Barber Steamship Lines, Inc. and Wilhelm Wilhelmsen also contested
the claim alleged that the carrier's liability for the shipment ceased upon discharge thereof
from the ship's tackle; 

that they and their co-defendant Manila Port Service are not the agents of the vessel;

that the said 218 packages were discharged from the vessel now, are in the custody of Manila
Port Service as operator of the arrastre service for the Port of Manila;

that such damage was caused by insufficiency of packing, force majeure and/or perils of the
sea;

and that they offered to settle the latter's claim in full by paying the C.I.F. value of 27 lbs.
caramel 4.13 kilos methyl salicylate and 12 pieces pharmaceutical vials of the shipment, but
their offer was declined by the consignee and/or the plaintiff.

After due trial, the lower court, on March 10, 1965 rendered judgment ordering defendants
Macondray & Co., Inc., Barber Steamship Lines, Inc. and Wilhelm Wilhelmsen to pay to the
plaintiff, jointly and severally, the sum of P300.00, with legal interest thereon from the filing of
the complaint until fully paid, and defendants Manila Railroad Company and Manila Port
Service to pay to plaintiff, jointly and severally, the sum of P809.67, with legal interest
thereon from the filing of the complaint until fully paid, the costs to be borne by all the said
defendants. 
On April 12, 1965, plaintiff, contending that it should recover the amount of $1,134.46, or its
equivalent in pesos at the rate of P3.90, instead of P2.00, for every US$1.00, filed a motion for
reconsideration, but this was denied by the lower court on May 5, 1965. Hence, the present
appeal.

Plaintiff-appellant argues that, as subrogee of the consignee, it should be entitled to recover


from the defendants-appellees the amount of $1,134.46 which it actually paid to the
consignee and which represents the value of the lost and damaged shipment as well as other
legitimate expenses such as the duties and cost of survey of said shipment, and that the
exchange rate on the date of the judgment, which was P3.90 for every US$1.00, should have
been applied by the lower court.

Defendants-appellees countered that their liability is limited to the C.I.F. value of the goods,
pursuant to contract of sea carriage embodied in the bill of lading that the consignee's
(Winthrop-Stearns Inc.) claim against the carrier (Macondray & Co., Inc., Barber Steamship
Lines, Inc., Wilhelm Wilhelmsen and the arrastre operators (Manila Port Service and Manila
Railroad Company) was only for the sum of Pl,109.67, representing the C.I.F. value of the loss
and damage sustained by the shipment which was the amount awarded by the lower court to
the plaintiff-appellant; defendants appellees are not insurers of the goods and as such they
should not be made to pay the insured value therefor; the obligation of the defendants-
appellees was established as of the date of discharge, hence the rate of exchange should be
based on the rate existing on that date, i.e., August 7, 1960,  and not the value of the currency
at the time the lower court rendered its decision on March 10, 1965.

ISSUE:

Is the carrier, in case of loss or damage, liable to the consignee is limited to the C.I.F. value of
the goods which were lost or damaged. (Yes)

RULING:

The appeal is without merit.

The purpose of the bill of lading is to provide for the rights and liabilities of the parties in
reference to the contract to carry. The stipulation in the bill of lading limiting the common
carrier's liability to the value of the goods appearing in the bill, unless the shipper or owner
declares a greater value, is valid and binding.

This limitation of the carrier's liability is sanctioned by the freedom of the contracting parties
to establish such stipulations, clauses, terms, or conditions as they may deem convenient,
provided they are not contrary to law, morals, good customs and public policy. 

A stipulation fixing or limiting the sum that may be recovered from the carrier on the loss or
deterioration of the goods is valid, provided it is (a) reasonable and just under the
circumstances, and (b) has been fairly and freely agreed upon.

The plaintiff-appellant, as insurer, after paying the claim of the insured for damages under the
insurance, is subrogated merely to the rights of the assured. As subrogee, it can recover only
the amount that is recoverable by the latter. Since the right of the assured, in case of loss or
damage to the goods, is limited or restricted by the provisions in the bill of lading, a suit by
the insurer as subrogee necessarily is subject to like limitations and restrictions.
In the case at bar, the liabilities of the defendants- appellees with respect to the lost or
damaged shipments are expressly limited to the C.I.F. value of the goods as per contract of
sea carriage embodied in the bill of lading, which reads:

Whenever the value of the goods is less than $500 per package or other freight
unit, their value in the calculation and adjustment of claims for which the
Carrier may be liable shall for the purpose of avoiding uncertainties and
difficulties in fixing value be deemed to be the invoice value, plus frieght and
insurance if paid, irrespective of whether any other value is greater or less.

The limitation of liability and other provisions herein shall inure not only to the
benefit of the carrier, its agents, servants and employees, but also to the
benefit of any independent contractor performing services including
stevedoring in connection with the goods covered hereunder. (Paragraph 17,
emphasis supplied.)

It is not pretended that those conditions are unreasonable or were not freely and fairly agreed
upon. The shipper and consignee are, therefore, bound by such stipulations since it is
expressly stated in the bill of lading that in "accepting this Bill of Lading, the shipper, owner
and consignee of the goods, and the holder of the Bill of Lading agree to be bound by all its
stipulations, exceptions and conditions, whether written, stamped or printed, as fully as if
they were all signed by such shipper, owner, consignee or holder. It is obviously for this
reason that the consignee filed its claim against the defendants-appellees on the basis of the
C.I.F. value of the lost or damaged goods in the aggregate amount of Pl,109.67

WHEREFORE, the appealed decision is hereby affirmed, with costs against the plaintiff-
appellant.

National Union Fire Insurance v. Stolt Nielsen, GR 87958, 26 April 1990


FACTS: 
1985: United Coconut Chemicals, the shipper/assured.. shipped 404 metric tons of distilled fatty acid
to Niewe Matex in Netherlands.
 
From Batangas to Netherlands, the shipment was aboard MT Stolt Sceptre, a tanker owned by Stolt-
Nielsen Philippines, (or the carrier). Stolt Nielsen issued a Bill of Lading, which contained a general
statement of incorporation of the terms of a Charter Party, between SHIPPER and CARRIER.
 
Petitioner National Union Fire Insurance Company (NUFIC), a non-life American insurance
corporation, insured the shipment under a marine cargo policy…
 
The shipment arrived in Netherlands, and it was found to be discolored and totally contaminated!
 
The SHIPPER/ASSURED filed a claim against the CARRIER.. but the latter denied.
> Hence, the INSURER indemnified the SHIPPER/ASSURED.
 
As subrogee of the SHIPPER-ASSURED, the INSURER filed suit against the CARRIER, before
the RTC… for recovery of the sum of P1,619,469.21, with interest,
> representing the amount the INSURER had paid the SHIPPER-ASSURED.
 
RTC dismissed the case, and the CA ordered that the INSURER refer its claims for arbitration; and
respondent Judge is directed to suspend the proceedings in Civil case pending the return of the
arbitral award.
INSURER argues: > that it is not bound by the Charter Party because as insurer, it is subrogee only
with respect to the Bill of Lading; that only the Bill of Lading should regulate the relation among the
INSURER, the holder of the Bill of Lading, and the CARRIER; THAT in order to bind it, the arbitral
clause in the Charter Party should have been incorporated into the Bill of Lading.
 
ISSUE FOR THE SC: Are the terms of the Charter Party, particularly the provision on arbitration,
binding on the INSURER?

HELD: SC RULES AGAINST THE INSURER.


      > the INSURER cannot avoid the binding effect of the arbitration clause.
The Bill of Lading in issue provides that:
“This shipment is carried under and pursuant to the terms of the Charter…all the
terms whatsoever of the said Charter except the rate and payment of freight specified
therein apply to and govern the rights of the parties concerned in this shipment…” 
The provision on arbitration in the Charter Party reads:
“4. Arbitration. Any dispute arising from the making, performance or termination of this
Charter Party shall be settled in New York, Owner and Charterer each appointing an
arbitrator,…”
 
Clearly, the Bill of Lading incorporates by reference the terms of the Charter Party.
> the charter may be made part of the contract through an appropriate reference in the Bill of Lading
…>> This should include the provision on arbitration even without a specific stipulation to that effect.
The entire contract must be read together and its clauses interpreted in relation to one another and
not by parts.
 
In some cases where a Bill of Lading has been issued by a carrier covering goods shipped aboard a
vessel under a charter party, and the charterer is also the holder of the bill of lading, "the bill of
lading operates as the receipt for the goods, and as document of title passing
the property of the goods, but not as varying the contract between the charterer and the shipowner"
…The Bill of Lading becomes, therefore, only a receipt and not the contract of carriage in a charter
of the entire vessel, for the contract is the Charter Party the law between the parties who are bound
by its terms and condition provided that these are not contrary to law, morals, good customs, public
order and public policy (Article 1306, Civil Code).
 
CA found correctly: the INSURER "cannot feign ignorance of the arbitration clause since it was
already charged with notice of the existence of the charter party due to an appropriate reference in
the bill of lading and, by the exercise of ordinary diligence, it could have easily obtained a copy
thereof either from the shipper or the charterer.
 
> By subrogation, it became privy to the Charter Party as fully as the SHIPPER before the
latter was indemnified, because as subrogee, it stepped into the shoes of the SHIPPER-
ASSURED and is subrogated merely to the latter's rights. 
> It can recover only the amount that is recoverable by the assured.
> And, since the right of action of the SHIPPER-ASSURED is governed by the provisions of
the Bill of Lading, which includes by reference the terms of the Charter Party, necessarily, a
suit by the INSURER is subject to the same agreements
 
⇒ as the subrogee of the SHIPPER, the INSURER is contractually bound by the terms of the
Charter party… Any claim of inconvenience or additional expense on its part should not render the
arbitration clause unenforceable.
 
→ Referral to arbitration in New York, and suspension of the proceedings in Civil Case, pending the
return of the arbitral award, is called for. >> hence, no reversible error in CA decision
→ Petition for Review on certiorari is DENIED and the said judgment is hereby AFFIRMED.
 
 
 
 

***

Cebu Shipyard v William Lines, GR 132607, 5 May 1999- Cruz

Cebu Shipyard v William, G.R. No. 132607. May 5, 1999


FACTS: Cebu Shipyard and Engineering Works, Inc. repaired marine vessels while the Prudential is
in the non-life insurance business. William Lines, Inc., the owner of M/V Manila City, a luxury
passenger-cargo vessel, which caught fire and sank. At the time of the incident, the subject vessel
was insured with Prudential for P45M for hull and machinery. CSEW was insured for only Php 10
million for the shiprepairer’s liability policy. They entered into a contract where negligence was the
only factor that could make CSEW liable for damages. Moreover, liability of CSEW was limited to
only Php 1 Million for damages. The Hull Policy included an “Additional Perils (INCHMAREE)”
Clause covering loss of or damage to the vessel through the negligence of, among others, ship
repairmen. William brought Manila City to the dry dock of CSEW for repairs. The officers and cabin
crew stayed at the ship while it was being repaired. After the vessel was transferred to the docking
quay, it caught fire and sank, resulting in its total loss. William brought suit against CSEW alleging
that it was through the latter’s negligence that the ship caught fire and sank. Prudential was
implemented as co-plaintiff after it had paid the value of insured items. It was subrogated to 45
million, or the value it claimed to indemnify. The trial court brought judgment against CSEW 45
million for the ship indemnity, 65 million for loss of income, and more than 13 million in other
damages. The CA affirmed the trial court decision.CSEW contended that the cause of the fire was
due to William’s hot works on the said portion of the ship which they didn’t ask CSEW permission
for.
 Prudential, on the other hand, blamed the negligence of the CSEW workers in the instance when
they didn’t mind rubber insulation wire coming out of the air-conditioning unit that was already
burning. Hence this MFR.
 
ISSUE:  Whether or not Prudential has the right of subrogation against its own insured.
 
RATIO/HELD: No, Petitioner contends that Prudential is not entitled to be subrogated to the rights of
William Lines, Inc., theorizing that (1) the fire which gutted M/V Manila City was an excluded risk and
(2) it is a co-assured under the Marine Hull Insurance Policy. This was wrong. The one who caused
the fire has already been adjudicated by the courts as CSEW. Upon proof of payment by Prudential
to William Lines, Inc., the former was subrogated to the right of the latter to indemnification from
CSEW.  As aptly ruled by the Court of Appeals, the law says: “Art. 2207. If the plaintiff’s property has
been insured, and he has received indemnity from the insurance company for the injury or loss
arising out of the wrong or breach of contract complained of, the insurance company shall be
subrogated to the rights of the insured against the wrongdoer or the person who has violated the
contract. If the amount paid by the insurance company does not fully cover the injury or loss, the
aggrieved party shall be entitled to recover the deficiency from the person causing the loss or injury.”
When Prudential paid the latter the total amount covered by its insurance policy, it was subrogated
to the right of the latter to recover the insured loss from the liable party, CSEW. Petitioner theorizes
further that there can be no right of subrogation as it is deemed a co-assured under the subject
insurance policy with reliance on Clause 20 of the Work Order which states: 20. The insurance on
the vessel should be maintained by the customer and/or owner of the vessel during the period the
contract is in effect. Clause 20 of the Work Order in question is clear in the sense that it requires
William Lines to maintain insurance on the vessel during the period of dry-docking or repair. 
However, the fact that CSEW benefits from the said stipulation does not automatically make it as a
co-assured of William Lines. The intention of the parties to make each other a co-assured under an
insurance policy is to be read from the insurance contract or policy itself and not from any other
contract or agreement because the insurance policy denominates the beneficiaries of the insurance. 
The hull and machinery insurance procured by William Lines, Inc. from Prudential named only
“William Lines, Inc.” as the assured. There was no manifestation of any intention of William Lines,
Inc. to constitute CSEW as a co-assured under subject policy.  The claim of CSEW that it is a co-
assured is unfounded. Then too, in the Additional Perils Clause of the same Marine Insurance
Policy, it is provided that this insurance also covers loss of or damage to vessel directly caused by
the negligence of charterers  and repairers who are not assured. As correctly pointed out by
respondent Prudential, if CSEW were deemed a co-assured under the policy, it would nullify any
claim of William Lines, Inc. from Prudential for any loss or damage caused by the negligence of
CSEW. Certainly, no shipowner would agree to make a ship repairer a co-assured under such
insurance policy; otherwise, any claim for loss or damage under the policy would be invalidated. 
 

Manila Mahogany v CA, GR L-52756, 12 October 1987- Benito

Manila Mahogany v. CA

GR L-52756

October 12, 1987

Topics: Subrogation; Insurer’s right to recover from the insured

Facts: Manila Mahogany Manufacturing insured its Mercedez Benz 4-door sedan with private
respondent, Zenith Insurance Corporation. In between the period covered by the insurance, the
insured vehicle was bumped and damaged by a truck owned by San Miguel Corporation. Zenith
then paid Manila Mahogany P5,000 for the damage caused. The general manager of Manila
Mahogany executed a Release of Claim, subrogating Zenith all its right to action against San
Miguel. When Zenith demanded reimbursement from San Miguel thru Insurance Adjusters, Inc.,
it was refused to be reimbursed in the amount it paid to Manila Mahogany. It now demanded
reimbursement from the assured for P4,500, the amount paid to San Miguel. However, the
insured refused to pay. Thus, Zenith filed a suit.

Issue: Whether or not the insurance company is entitled to recover from the assured the
amount of insurance money paid on account of right of subrogation
Held/Ratio: Yes, the insurer Zenith Insurance may recover the insurance money in the amount
of P5,000 paid to the assured, Manila Mahogany. As held in Phil Air Lines v. Heald Lumber Co.,
if a property is insured and the owner receives the indemnity from the insurer, it is provided in
Art. 2207 of the New Civil Code that the insurer is deemed subrogated to the rights of the
insured against the wrongdoer, and if the amount paid by the insurer does not fully cover the
loss, then the aggrieved party is the one entitled to recover the deficiency. Although petitioner’s
right to file deficiency claim against San Miguel is with legal basis, without prejudice to the
insurer’s right of subrogation, nevertheless when it (Manila Mahogany) executed another
release claim discharging San Miguel from all actions arising out of the consequence of the
accident after the insurer had paid the proceeds of the policy, the insurer is entitled to recover
from the insured the amount of insurance money paid. 

F.F. Cruz v CA, GR L-52732, 29 August 1988- SARREAL

F.F. Cruz and Co., Inc.


vs
The Court of Appeals

G.R. No. L-52732


August 29,1998

Facts:
The furniture manufacturing shop of the petitioner in Caloocan City was situated adjacent to the
residence of private respondents. Sometime in August 1971, private respondent Gregorio Mable
first approached Eric Cruz, petitioner's plant manager, to request that a firewall be constructed
between the shop and private respondents' residence. The request was repeated several times but
they fell on deaf ears. In the early morning of September 6, 1974, fire broke out in the
petitioner's shop. Petitioner's employees, who slept in the shop premises, tried to put out the fire,
but their efforts proved futile. The fire spread to the private respondents' house. Both the shop
and the house were razed to the ground. The cause of the conflagration was never discovered.
The National Bureau of Investigation found specimens from the burned structures negative for
the presence of inflammable substances. Subsequently, private respondents collected P35,000.00
on the insurance on their house and the contents thereof.

On January 23, 1975, private respondents filed an action for damages against the petitioner,
praying for a judgment in their favor awarding P150,000.00 as actual damages, P50,000.00 as
moral damages, P25,000.00 as exemplary damages, P20,000.00 as attorney's fees and costs. The
Court of First Instance(Regional Trial Court) renders judgement in favor of respondents. The
Court of Appeals affirmed the decision of the Regional Trial Court but reduced the award of
damages. The petitioner contends that the Court of Appeals erred in not deducting the sum of
35,000, which private respondents recovered from insurance of their house, from the award of
damages. Hence, the petitioner filed the instant petition for review.
Issue: 

Whether or not the respondents may still recover from petitioner notwithstanding the indemnity
paid by the insurer. 

Held: 

Yes, the respondents may still recover from the petitioner if the amount paid by the insurance
company does not fully cover the injury or loss. Under Article 2207 of the Civil Code of the
Philippines, if the plaintiffs property has been insured, and he has received indemnity from the
insurance company for the injury or loss arising out of the wrong or breach of contract
complained of, the insurance company shall be  subrogated to the rights of the insured against
the wrongdoer or the person who violated the contract. If the amount paid by the insurance
company does not fully cover the injury or loss, the aggrieved party shall be entitled to recover
the deficiency from the person causing the loss or injury. Article 2207 of the Civil Code is
founded on the well settled principle of subrogation. If the insured property is destroyed or
damaged through the fault or negligence of a party other than the assured, then the insurer, upon
payment to the assured, will be subrogated to the rights of the assured to recover from the
wrongdoer to the extent that the insurer has been obligated to pay. The law is clear and needs no
interpretation. The fact that private respondents have been indemnified by their insurer in the
amount of P35,000.00 for the damage caused to their house and its contents has not escaped the
attention of the Court. Hence, the Supreme Court affirmed the decision of the Court of Appeals
with modifications as to the damages awarded for the loss of private respondents' house,
considering their receipt of P35,000.00 from their insurer: (1) the damages awarded for the loss
of the house is reduced to P35,000.00; and (2) the right of the insurer to subrogation and thus
seek reimbursement from petitioner for the P35,000.00 it had paid private respondents is
recognized.

H. Construction of insurance contracts


De la Cruz v. Capital Insurance, GR L-21574, 30 June 1966-

SIMON DE LA CRUZ, plaintiff and appellee, vs. THE CAPITAL INSURANCE &
SURETY Co., lNC., defendant and appellant.

FACTS:
Eduardo de la Cruz, a non-professional boxer, had an accident insurance policy with
Capital Insurance, with his father Simon, as the beneficiary. For the New Year’s
celebration, he joined a boxing contest. In the course of the match, Eduardo
unintentionally slipped and was hit by his opponent on the left part of the back of the
head, causing Eduardo to fall, with his head hitting the rope of the ring. He died
because of cranial injury sustained. Simon filed a claim with Capital insurance but the
latter refused on the ground that Eduardo’s death was not by “accidental means,” as
Eduardo voluntarily joined the boxing contest, and that such participation was the
“means” contemplated under the policy for which he was insured “against death or
disability caused by accidental means.

Simon filed an action for specific performance with the CFI. 

The CFI ruled in favor of de la Cruz. The SC affirmed. 

ISSUE:
Should Capital Insurance be exempted from liability under the policy on the ground that
Eduardo’s death was not by “accidental means”? 

Ruling:

NO.

“Accident” and “accidental” mean that which happen by chance or fortuitously,


without intention and design, and which is unexpected, unusual, and unforeseen.
As used in insurance policies, the trend is to consider these terms legally synonymous
with “accidental means.” The insurer is liable where the death or injury is a natural or
probable result of the insured’s voluntary act, or if something unforeseen occurs in
the doing of the act which produces the injury, the resulting death is within the
protection of policies insuring against death or injury from accident.

In this case, while Eduardo voluntarily joined the boxing contest, the injury was
sustained when he slid, giving occasion to the infliction by his opponent of the blow that
threw him to the ropes of the ring. This unintentional slipping was described by the
Court as an “unfortunate incident.” The Court also noted that is physically
demanding sports as boxing, death is not ordinarily expected as a result unless
some accident or unforeseen event intervened. 

Furthermore, the policy involved herein specifically excluded from its coverage—

(e) Death or disablement consequent upon the Insured engaging in football, hunting,
pigsticking, steeplechasing, poloplaying, racing of any kind, mountaineering, or
motorcycling.”

Death or disablement resulting from engagement in boxing contests was not


declared outside of the protection of the insurance contract. Failure of Capital
Insurance to include death resulting from a boxing match or other sports among
the prohibitive risks leads inevitably to the conclusion that it did not intend to
limit or exempt itself from liability for such death.
Judgment affirmed. 

Sun Insurance Office Ltd v CA, GR 92383, 17 July 1992-

SUN INSURANCE V. CA

G.R. No. 92383

July 17, 1992

FACTS:

Felix Lim availed an insurance policy with the petitioner. Two months later, he was dead with a
bullet wound in his head. As beneficiary, his wife Nerissa Lim sought payment on the policy but
her claim was rejected. Pilar Nalagon, Lim's secretary, was the only eyewitness to his death. It
happened on October 6, 1982, at about 10 o'clock in the evening, after his mother's birthday
party. According to Nalagon, Lim was in a happy mood (but not drunk) and was playing with his
handgun, from which he had previously removed the magazine. As she watched television, he
stood in front of her and pointed the gun at her. She pushed it aside and said it might he loaded.
He assured her it was not and then pointed it to his temple. The next moment there was an
explosion and Lim slumped to the floor. He was dead before he fell.
The widow sued the petitioner.
The petitioner also cites one of the four exceptions provided for in the insurance contract and
contends that the private petitioner's claim is barred by such provision. It is there stated:
Exceptions —
The company shall not be liable in respect of

1. Bodily injury
xxx xxx xxx
b. consequent upon
i) The insured person attempting to commit suicide or willfully exposing himself to
needless peril except in an attempt to save human life.

 
RTC: In favor of the widow. Ordered the petitioner to pay the amount of the policy.
CA: AFFIRMED

ISSUE:

Whether the widow is entitled to the policy.

RULING:

Yes.
It should be noted at the outset that suicide and willful exposure to needless peril are in pari
materia because they both signify a disregard for one's life. The only difference is in degree, as
suicide imports a positive act of ending such life whereas the second act indicates a
reckless risking of it that is almost suicidal in intent. To illustrate, a person who walks a
tightrope one thousand meters above the ground and without any safety device may not actually
be intending to commit suicide, but his act is nonetheless suicidal. He would thus be
considered as "willfully exposing himself to needless peril" within the meaning of the
exception in question.

The words "accident" and "accidental" have never acquired any technical signification in law,
and when used in an insurance contract are to be construed and considered according to
the ordinary understanding and common usage and speech of people generally. In-
substance, the courts are practically agreed that the words "accident" and "accidental" mean
that which happens by chance or fortuitously, without intention or design, and which is
unexpected, unusual, and unforeseen. 

In this case there is accident.

Ty v Filipinas Cia de Seguros, GR L-21821-22 and L-21824-27, 31 May 1966 

Facts:
Ty was employed as a mechanic operator by Braodway Cotton Factory at Grace Park,
Caloocan. In 1953, he took personal accident policies from 7 insurance companies
(defendants), on different dates, effective for 12 mos. On Dec. 24. 1953, a fire broke out in the
factory where Ty was working.  A heavy object fell on his hand when he was trying to put
out the fire. Ty received treatment at the National Orthopedic Hospital for six listed injuries. 
The attending surgeon certified that these injuries would cause the temporary total
disability of Ty’s left hand. 

Insurance companies refused to pay Ty’s claim for compensation under the policies by
reason of said disability of his left hand.  Ty filed a complaint in the municipal court who
decided in his favor. CFI reversed on the ground that under the uniform terms of the policies,
partial disability due to loss of either hand of the insured, to be compensable must be the result
of amputation.

Issue:
Whether or not Ty should be indemnified under his accident policies.

Held:
NO. The Court already ruled in the case of Ty v. FNSI that where the insurance policies
define partial disability as loss of either hand by amputation through the bones of the
wrist, the insured cannot recover under said policies for temporary disability of his left
hand caused by the fractures of some fingers.  The provision is clear enough to inform the
party entering into that contract that the loss to be considered a disability entitled to
indemnity, must be severance or amputation of the affected member of the body of the
insured.
Calanoc v CA, GR 8151, 16 December 1955 - Buan

VIRGINIA CALANOC, petitioner,

vs.

COURT OF APPEALS and THE PHILIPPINE AMERICAN LIFE INSURANCE CO.,


respondents.

INSURANCE LAW; ACCIDENTAL DEATH; AMBIGUOUS TERMS IN INSURANCE POLICY.

While as a general rule “the parties may limit the coverage of the policy to certain particular
accidents and risks or causes of loss, and may expressly except other risks or causes of loss
therefrom”

However, it is to be desired that the terms and phraseology of the exception clause be
clearly expressed so as to be within the easy grasp and understanding of the insured, for
if the terms are doubtful or obscure the same must of necessity be interpreted or
resolved against the one who has caused the obscurity. (Article 1377, new Civil Code.)

 FACTS:

 Melencio Basilio was a watchman of the Manila Auto Supply located at the corner of
Avenida Rizal and Zurbaran. He secured a life insurance policy from the Philippine
American Life Insurance Company in the amount of P2,000 to which was attached a
supplementary contract covering death by accident.

  On January 25, 1951 while Melencio Basilio was on duty as watchman of the Manila
auto Supply, a neighbor in the name of Atty. Antonio Ojeda approaches him stating
that he has suspicion that his house was being rob. Atty. Ojeda asks Basilio to
accompany him but the latter refuses on the ground that was not a policeman.

  Atty Ojeda asked a traffic policeman to accompany him, to which the policeman agreed.
While Ojeda and the policeman pass by Basilio, they somehow invite Basilio to come
along. As the three approached the gate of Ojeda’s residence, a shot was fired hitting
Basilio in the abdomen resulting to his death.
  Virginia Calanoc, the widow, was paid the sum of P2,000, face value of the policy, but
when she demanded the payment of the additional sum of P2,000 representing the
value of the supplemental policy, the company refused alleging, as main defense, that
the deceased died because he was murdered by a person who took part in the
commission of the robbery and while making an arrest as an officer of the law
which contingencies were expressly excluded in the contract and have the effect of
exempting the company from liability.

 ISSUE:

Whether the Philippine American Insurance is liable for the supplemental contract?

 HELD:
Yes. The circumstance that he was a mere watchman and had no duty to heed the call of Atty.
Ojeda should not be taken as a capricious desire on his part to expose his life to danger
considering the fact that the place he was duty-bound to guard was only a block away.

In volunteering to extend help under the situation, he might have thought, rightly or wrongly,
that to know the truth was in the interest of his employer, it being a matter that affects
the security of the neighborhood. No doubt there was some risk coming to him in pursuing
that errand, but that risk always existed, it being inherent in the position he was holding.

He cannot therefore be blamed solely for doing what he believed was in keeping with his duty
as a watchman and as a citizen. And he cannot be considered as making an arrest as an officer
of the law, as contended, simply because he went with the traffic policeman, for certainly he did
not go there for that purpose nor was he asked to do so by the policeman.

While as a general rule “the parties may limit the coverage of the policy to certain particular a
and risks or causes of loss, and may expressly except other risks or causes of loss therefrom”
however, it is to be desired that the terms and phraseology of the exception clause be
clearly expressed so as to be within the easy grasp and understanding of the insured, for
if the terms are doubtful or obscure the same must of necessity be interpreted or
resolved against the one who has caused the obscurity.

Biagtan v Insular Life, GR L-25579, 29 March 1972- CRUZ

 BIAGTAN v. INSULAR LIFE, GR No. L-25579, 1972-03-29


FACTS: 
Juan S. Biagtan was insured with defendant Insular Life Assurance Company under Policy No.
398075 for the sum of P5,000.00 and, under a supplementary contract denominated "Accidental
Death Benefit Clause", for an additional sum of P5,000.00 *if "the death of the Insured
resulted directly from bodily injury effected solely through external and violent means
sustained in an accident * * * and independently of all other causes." The clause,
however, expressly provided that it would not apply where death resulted from an injury
"intentionally inflicted by a third party."

On the night of May 20, 1964 or during the first hours of the following day a band of robbers
entered the house of the insured Juan S. Biagtan.

that in committing the robbery, the robbers, on, reaching the staircase landing of the second
floor, rushed towards the doors of the second floor room, where they suddenly met a person
near the door of one of the... rooms who turned out to be the insured Juan S. Biagtan who
received thrusts from their sharp-pointed instruments, causing wounds on the body of said
Juan S. Biagtan resulting in his death at about 7 a.m. on the same day, May 21, 1964

Plaintiffs, as beneficiaries of the insured, filed a claim under the policy.

The insurance company paid the basic amount of P5,000.00 but refused to pay the additional
sum of P5,000.00 under the accidental death benefit clause, on the ground that the
insured's death... resulted from injuries intentionally inflicted by third parties and
therefore was not covered.
Plaintiffs filed suit to recover, and after due hearing the court a quo rendered judgment in their
favor.

ISSUE:
Whether under the facts as stipulated and found by the trial court the wounds received by the
insured at the hands of the robbers nine in all, five of them mortal and four non-mortal were
inflicted intentionally.

RULING:
The case of Calanoc vs. Court of Appeals, 98 Phil. 79, is relied upon by the trial court in support
of its decision.  The facts in that case, however, are different from those obtaining here.

For while a single shot fired from a distance, and by a person who was not even seen... aiming
at the victim, could indeed have been fired without intent to kill or injure, nine wounds inflicted
with bladed weapons at close range cannot conceivably be considered as innocent insofar as
such intent is concerned.  The manner of execution of the crime permits no... other conclusion.

Thus, it has been held that "intentional" as used in an accident policy excepting... intentional
injuries inflicted by the insured or any other person, etc., implies the exercise of the reasoning
faculties, consciousness, and volition.[1] Where a provision of the policy excludes intentional
injury, it is the intention of the person... inflicting the injury that is controlling.[2]   If the injuries
suffered by the insured clearly resulted from the intentional act of a third person the insular is
relieved from liability as stipulated.

Finman General Assurance v CA, GR 100970, 2 September 1992-

QUA CHEE GAN, plaintiff-appellee, vs. LAW UNION AND ROCK INSURANCE CO., LTD.,
defendant-appellant., G.R. No. L-4611, December 17, 1955 - ALMOCERA

 FACTS:

Plaintiff-appellee owned four bodegas used for the storage of copra and hemp. These buildings,
together with their contents, were insured with the defendant company since 1937 and the loss
made payable to the Philippine National Bank as mortgage of the hemp and crops, to the extent
of its interest.

Sometime after, three of the bodegas, together with the merchandise inside, were completely
destroyed by fire of an undetermined origin. Consequently, Qua Chee Gan notified the
insurance company of his loss. The latter conducted an extensive   investigation. The damage
was determined to be equivalent to P398,562.81 which was later reduced to the full amount of
the insurance, Php370,000.00. However, the insurance company refused payment, claiming
violation of warranties and conditions, filing of fraudulent claims, and that the fire had been
deliberately caused by the insured or by other persons in connivance with him. If moreover
argued that since the bodegas insured   had an external wall perimeter of 500 meters or 1,640
feet, the appellee should have 11 fire hydrants in the compound, and that he actually had only 2
with a further pair nearby, belonging to the municipality of Tabaco.

ISSUE:

Whether or not the insurer company is liable.

RULING:

YES. The SC is in agreement with the trial court that the appellant is barred by waiver (or  
rather estoppel) to claim violation of the so-called fire hydrants warranty, for the reason that
knowing fully all that the number of hydrants demanded therein never existed from the very
beginning, the appellant nevertheless issued the policies in question subject to   such warranty,
and received the corresponding premiums. It would be perilously close to conniving at fraud
upon the insured to allow appellant to claims now as void ab initio the policies that it had issued
to the plaintiff without warning of their fatal defect, of which it was informed, and after it had
misled the defendant into believing that the policies were effective. The contract of insurance is
one of perfect good faith (uferrimal fidei) not for the insured alone, but equally so for the insurer,
in fact, it is mere so for the latter, since its dominant bargaining position carries with it stricter
responsibility. We find no reversible error in the judgment appealed from, wherefore the same is
hereby affirmed. Costs against the appellant. Estoppel due to insurer’s inequitable conduct.

G.R. No. L-16215             June 29, 1963

SIMEON DEL ROSARIO, plaintiff-appellee,


vs.
THE EQUITABLE INSURANCE AND CASUALTY CO., INC., defendant-appellant.
 
 
FACTS:
-April 13, 1957: Simeon del Rosario, father of the insured who died from drowning filed
a claim for payment with Equitable Ins. and Casualty Co., Inc. but it refused to pay more
than P1,000 php so a case was filed with the RTC for the P2,000 balance stating that
under the policy they are entitled to P1,000 to P3,000 as indemnity
- RTC: entitled to recover P3,000 - policy does not positively state any definite amount,
there is an ambiguity in this respect in the policy, which ambiguity must be interpreted in
favor of the insured and strictly against the insurer so as to allow greater indemnity
 

ISSUE: W/N Simeon is entitled to recover P3,000 


HELD: YES.
-  terms in an insurance policy, which are ambiguous, equivocal or uncertain are to be
construed strictly against, the insurer, and liberally in favor of the insured so as to effect
the dominant purpose of indemnity or payment to the insured, especially where a
forfeiture is involved
-  reason for this rule is that the "insured usually has no voice in the selection or
arrangement of the words employed and that the language of the contract is selected
with great care and deliberation by expert and legal advisers employed by, and acting
exclusively in the interest of, the insurance company

Verendia v CA, GR 75605, 22 January 1993 - DATILES

DOCTRINE:
An insurance contract, which is a contract of adhesion, should be liberally construed in favor
of the insured and strictly against the insurer company, which usually prepares it. However,
when the insured acts in bad faith, the terms of the policy should be strictly against the insured.
 
FACTS:
Fidelity and Surety Insurance Company of the Philippines (Fidelity) issued a Fire Insurance
Policy to cover Rafael (Rex) Veranda’s residence from June 23, 1980 til June 23, 1981 in the
amount of P385,000.00. Monte de Piedad & Savings Bank. The residential building was also
insured by two other companies: Country Bankers Insurance for P56,000.00 and Development
Insurance for P400,000.00. The property was completely burned down on December 28, 1980.
Fidelity was informed of the loss but refused to pay despite demands.
 
Verendia filed a complaint before the trial court, praying for payment of P385,000.00. The
complaint was amended to include Monte de Piedad as an “unwilling defendant”. Fidelity
argued that the policy was avoided due to over-insurance and that Verendia maliciously
represented that the building at the time of the fire was leased under contract to Roberto
Garcia when it was in fact Marcelo Garcia who was lessee.
 
The trial court ruled in favor of Fidelity by agreeing with its defenses, as well as Verendia’s
violation of Paragraph 3 of the policy, which provides that the insured must inform Fidelity
of his other insurance coverages, which he failed to do.

The case was appealed to the Intermediate Appealate Court, which reversed the trial court’s
decision: a) there was no misrepresentation concerning the lessee; b) Paragraph 3 was waived
by Fidelity when it attempted to settle the claims of Verendia.
 
ISSUE: WON the contract of lease submitted by Verendia to support his claim on the fire
insurance policy constitutes a false declaration, which would forfeit his benefits under Section
13 of the policy.
 
RULING: YES
 
The contract of lease upon which Verendia relies to support his claim for insurance benefits, was
entered into between him and one Robert Garcia, a couple of days after the effectivity of the
insurance policy. When the rented residential building was razed to the ground on December 28,
1980, it appears that Robert Garcia (or Roberto Garcia) was still within the premises. However,
according to the investigation report prepared by Pat. Eleuterio M. Buenviaje of the Antipolo
police, the building appeared to have "no occupant" and that Mr. Roberto Garcia was "renting on
the otherside (sic) portion of said compound." These pieces of evidence belie Verendia's
uncorroborated testimony that Marcelo Garcia, whom he considered as the real lessee, was
occupying the building when it was burned.
 
Moreover, Robert Garcia disappeared after the fire. It was only on October 9, 1981 that an
adjuster was able to locate him. Robert Garcia then executed an affidavit before the National
Intelligence and Security Authority (NISA) to the effect that he was not the lessee of
Verendia's house and that his signature on the contract of lease was a complete forgery.
Thus, on the strength of these facts, the adjuster submitted a report dated December 4, 1981
recommending the denial of Verendia's claim. As it is also a contract of adhesion, an insurance
contract should be liberally construed in favor of the insured and strictly against the insurer
company which usually prepares it.
 
Considering, however, the foregoing discussion pointing to the fact that Verendia used a
false lease contract to support his claim under the Fire Insurance Policy, the terms of the
policy should be strictly construed against the insured. Verendia failed to live by the terms of
the policy, specifically Section 13 thereof which is expressed in terms that are clear and
unambiguous, that all benefits under the policy shall be forfeited "If the claim be in any
respect fraudulent, or if any false declaration be made or used in support thereof, or if any
fraudulent means or devises are used by the Insured or anyone acting in his behalf to obtain any
benefit under the policy". Verendia, having presented a false declaration to support his claim for
benefits in the form of a fraudulent lease contract, he forfeited all benefits therein by virtue of
Section 13 of the policy in the absence of proof that Fidelity waived such provision. Worse yet,
by presenting a false lease contract, Verendia, reprehensibly disregarded the principle that
insurance contracts are uberrimae fidae and demand the most abundant good faith.
 
 
Fortune Insurance v CA, GR 115278, 23 May 1995 - pascual

Facts

Fortune Insurance and Surety Co., Inc. (Fortune Insurance) insured the produces Bank of the
Philippines (producers). Under its policy, the general exemption clause states that the 
company  shall  not  be  liable  if there be any  loss  caused  by  any  dishonest,
fraudulent  or  criminal  act  of  the  insured  or  any officer,  employee,  partner, 
director,  trustee  or authorized  representative  of  the  Insured whether  acting  alone  or 
in  conjunction  with othes.

The armoured car of producers Bank was on its way from Pasay to Makati when it got robbed
by Its driver, Benjamin Magalog, security guard saturnino atriaga,(an employee of PRC
management and Unicorn Security Services respectively). Both were accompanied by edelmer
bantique, reynaldo aquino and certain john doe. All of them were charged with violation of P.D.
532 (Anti-Highway Robbery Law).

producers Bank of the Philippines Filed its claim insurace but with no vail. Thus they filed a case
in the RTC. RTC is satisfied in the contention of the plaintiff that driver Magalong and guard
Atiga are not its “officer, employee, trustee or authorized representative at the time of the
robbery. They are assigned by PRC Management and by Unicorn Security which alone wields
the power to dismiss them.
RTC and CA held that there should be recovery under the Money, Security, and Payroll
Robbery policy it issued to the private respondent.

ISSUE:

Whether Fortune insurance is not liable because Magalong  and Atiga  were  employees  or
authorized representatives  of  Producers

Held:

Yes

SC reversed the decision, finding the insurance not liable in producers claim the that Magalong 
and Atiga  were   employees  or authorized representatives  of  Producers including them in the
exception under the policy of insurance claim because they acted as the clients representative.

A contract of insurance is a contract of adhesion, thus any ambiguity therein should be resolved
against the insurer should be construed liberally in favor of the insured and strictly against the
insurer. If the terms of the contract are clear and unambiguous, there is no room for construction
and such terms cannot be enlarged or diminished by judicial construction.

It should be noted that the insurance policy entered into by the parties is a theft or robbery
insurance policy which is a form of casualty insurance. Section 174 of the Insurance Code
provides: “Casualty insurance is insurance covering loss or liability arising from accident
or mishap, excluding certain types of loss which by law or custom are considered as
falling exclusively within the scope of insurance such as fire or marine. It includes, but is
not limited to, employer’s liability insurance, public liability insurance, motor vehicle
liability insurance, plate glass insurance, burglary and theft insurance, personal accident
and health insurance as written by non life insurance companies, and other substantially
similar kinds of insurance.”

The purpose of the exception is to guard against liability should the theft be committed by one
having unrestricted access to the property. The terms “service” and “employment” are generally
associated with the idea of selection, control, and compensation.

An insurance contract is a contract of indemnity upon the terms and conditions specified therein.
It is settled that the terms of the policy constitute the measure of the insurer’s liability. In the
absence of statutory prohibition to the contrary, insurance companies have the same rights as
individuals to limit their liability and to impose whatever conditions they deem best upon their
obligations not inconsistent with public policy.

there is a less evidence that the contracts between Producers and PRC Management
Systems and Unicorn Security Services are “labor-only” contracts. But even granting for
the sake of argument that these contracts were not “labor-only” contracts, and PRC
Management Systems and Unicorn Security Services were truly independent contractors, we
are satisfied that Magalong and Atiga were, in respect of the transfer of Producer’s money from
its Pasay City branch to its head office in Makati, its “authorized representatives” who served as
such with its teller Maribeth Alampay. Howsoever viewed, Producers entrusted the three with
the specific duty to safely transfer the money to its head office, with Alampay to be responsible
for its custody in transit; Magalong to drive the armored vehicle which would carry the money;
and Atiga to provide the needed security for the money, the vehicle, and his two other
companions.

For these particular tasks, the 3 acted as agents of Producers. A “representative” is defined as
one who represents or stands in the place of another; one who represents others or another in a
special capacity, as an agent, and is interchangeable with “agent.”

Therefore, Fortune is exempt from liability under the general exceptions clause of the insurance
policy. 

Mis amis Lumber v Capital Insurance, GR L-21380, 20 May 1966- pascual

Facts

Plaintiff-appellee Misamis Lumber Corporation, under its former name, Lanao Timber Mills, Inc.,
insured its Ford Falcon motor car for the amount of P14,000 with the defendant-appellant,
Capital Insurance & Surety Company, Inc. Par 4 of its policy provided that “the Insured may
authorize the repair of the Motor Vehicle necessitated by damage for which the Company may
be liable under this policy provided that: the estimated cost of such repair does not exceed the
authorized Repair Limit. that the authorized repair limit is P150.

The insured car, while traveling along Aurora Boulevard, passed over a water hole then hit a
hollow block lying alongside it. Thus, he crankcase and flywheel housing of the car broke. The
car was towed and repaired by Morosi Motors for P302.27.

the defendant-appellant refused to pay for the total cost of towage and repairs, thus a suit was
filed.

CFI finding plaintiff and against the defendant insurer. The defendant-appellant admits liability in
the amount of P150, but not for any excess thereof.

Issues

Whether the insurance is liable for the whole cost of repair amounting P302.27

Ruling

No.

There is no need to interpret because paragraph 4 of the policy is clear and specific and leaves
no room for interpretation.

The insurance contract may be rather onerous (onesided), but that in itself does not
justify the abrogation of its express terms, terms which the insured accepted or adhered
to and which is the law between the contracting parties. Finally, to require the insurer to
prove that the cost of the repairs ordered by the insured is unreasonable, as the appealed
decision does, when the insurer was not given an opportunity to inspect and assess the damage
before the repairs were made, strikes us as contrary to elementary justice and equity.
ordering the defendant-appellant Capital Insurance & Surety Company, Inc. to pay not more
than P150.00 to the plaintiff-appellee Misamis Lumber Corporation. 

I. The Business of Insurance 

White Gold Marine v Pioneer Insurance, GR 154514, 28 July 2005- Lasheras

G.R. No. 154514. July 28, 2005

WHITE GOLD MARINE SERVICES, INC. vs. PIONEER INSURANCE AND SURETY
CORPORATION AND THE STEAMSHIP MUTUAL UNDERWRITING ASSOCIATION
(BERMUDA) LTD.

FACTS:

·         White Gold Marine Services, Inc. (White Gold) procured a protection


and indemnity coverage for its vessels from The Steamship Mutual
Underwriting Association (Bermuda) Limited (Steamship Mutual) through
Pioneer Insurance and Surety Corporation (Pioneer).

·         Subsequently, White Gold was issued a Certificate of Entry and


Acceptance.Pioneer also issued receipts evidencing payments for the
coverage.

·         When White Gold failed to fully pay its accounts, Steamship Mutual
refused to renew the coverage.

·         Steamship Mutual thereafter filed a case against White Gold for


collection of sum of money to recover the latter’s unpaid balance.

·         White Gold on the other hand, filed a complaint before the Insurance
Commission claiming that Steamship Mutual violated Sections 186 and
187 of the Insurance Code, while Pioneer violated Sections 299,300 and
301 in relation to Sections 302 and 303, thereof.

INSURANCE COMMISSION: dismissed the complaint.

RATIONALE:
1) It said that there was no need for Steamship Mutual to secure a license because it
was not engaged in the insurance business. It explained that Steamship Mutual was a
Protection and Indemnity Club (P & I Club).

2) Likewise, Pioneer need not obtain another license as insurance agent and/or a
broker for Steamship Mutual because Steamship Mutual was not engaged in the
insurance business.

3) Moreover, Pioneer was already licensed, hence, a separate license solely as


agent/broker of Steamship Mutual was already superfluous.

CA: AFFIRMED

 In its decision, the appellate court distinguished between P & I Clubs vis-à-vis
conventional insurance. The appellate court also held that Pioneer merely acted as a
collection agent of Steamship Mutual.

Petitioner's Contention:

Petitioner insists that Steamship Mutual as a P & I Club is engaged in the insurance
business.

To buttress its assertion, it cites the definition of a P & I Club in Hyopsung Maritime Co.,
Ltd. v. Court of Appeals as "an association composed of shipowners in general who
band together for the specific purpose of providing insurance cover on a mutual basis
against liabilities incidental to shipowning that the members incur in favor of third
parties."

It stresses that as a P & I Club, Steamship Mutual’s primary purpose is to solicit and
provide protection and indemnity coverage and for this purpose, it has engaged the
services of Pioneer to act as its agent.

Respondent's Contention:

Respondents contend that although Steamship Mutual is a P & I Club, it is not engaged
in the insurance business in the Philippines. It is merely an association of vessel owners
who have come together to provide mutual protection against liabilities incidental to
shipowning.

Respondents aver Hyopsung is inapplicable in this case because the issue in Hyopsung
was the jurisdiction of the court over Hyopsung.
ISSUES:

1) W/N Steamship Mutual, a P & I Club, engaged in the insurance business in the
Philippines

2) W/N  Pioneer need a license as an insurance agent/broker for Steamship Mutual

RULING:

The Steamship Mutual Underwriting Association (Bermuda) Ltd., and Pioneer Insurance
and Surety Corporation are ORDERED to obtain licenses and to secure proper
authorizations to do business as insurer and insurance agent, respectively. The
petitioner’s prayer for the revocation of Pioneer’s Certificate of Authority and removal of
its directors and officers, is DENIED.

[No. 1 ISSUE]

DOING INSURANCE BUSINESS DEFINITION

Section 2(2) of the Insurance Code enumerates what constitutes "doing an insurance
business" or "transacting an insurance business". These are:

(a) making or proposing to make, as insurer, any insurance contract;

(b) making, or proposing to make, as surety, any contract of suretyship as a vocation


and not as merely incidental to any other legitimate business or activity of the surety;

(c) doing any kind of business, including a reinsurance business, specifically recognized
as constituting the doing of an insurance business within the meaning of this Code;

(d) doing or proposing to do any business in substance equivalent to any of the


foregoing in a manner designed to evade the provisions of this Code.

The same provision also provides, the fact that no profit is derived from the making of
insurance contracts, agreements or transactions, or that no separate or direct
consideration is received therefor, shall not preclude the existence of an insurance
business.

TEST
The test to determine if a contract is an insurance contract or not, depends on the
nature of the promise, the act required to be performed, and the exact nature of the
agreement in the light of the occurrence, contingency, or circumstances under which the
performance becomes requisite. It is not by what it is called.

INSURANCE DEFINITION

Basically, an insurance contract is a contract of indemnity. In it, one undertakes for a


consideration to indemnify another against loss, damage or liability arising from an
unknown or contingent event.

MARINE INSURANCE

In particular, a marine insurance undertakes to indemnify the assured against marine


losses, such as the losses incident to a marine adventure.

Section 99 of the Insurance Code enumerates the coverage of marine insurance.

MUTUAL INSURANCE COMPANY

Relatedly, a mutual insurance company is a cooperative enterprise where the members


are both the insurer and insured. In it, the members all contribute, by a system of
premiums or assessments, to the creation of a fund from which all losses and liabilities
are paid, and where the profits are divided among themselves, in proportion to their
interest.

Additionally, mutual insurance associations, or clubs, provide three types of coverage,


namely, protection and indemnity, war risks, and defense costs.

A P & I Club IS A MUTUAL INSURANCE ASSOCIATION

A P & I Club is "a form of insurance against third party liability, where the third party is
anyone other than the P & I Club and the members." By definition then, Steamship
Mutual as a P & I Club is a mutual insurance association engaged in the marine
insurance business.
 

DOING INSURANCE BUSINESS & NEED LICENSE

The records reveal Steamship Mutual is doing business in the country albeit without the
requisite certificate of authority mandated by Section 187 of the Insurance Code. It
maintains a resident agent in the Philippines to solicit insurance and to collect payments
in its behalf. We note that Steamship Mutual even renewed its P & I Club cover until it
was cancelled due to non-payment of the calls. Thus, to continue doing business here,
Steamship Mutual or through its agent Pioneer, must secure a license from the
Insurance Commission.

Since a contract of insurance involves public interest, regulation by the State is


necessary. Thus, no insurer or insurance company is allowed to engage in the
insurance business without a license or a certificate of authority from the Insurance
Commission.

PIONEER, AS AGENT/BROKER OF STEAMSHIP MUTUAL, NEED A SPECIAL


LICENSE

Pioneer is the resident agent of Steamship Mutual as evidenced by the certificate of


registration issued by the Insurance Commission. It has been licensed to do or transact
insurance business by virtue of the certificate of authority issued by the same agency.
However, a Certification from the Commission states that Pioneer does not have a
separate license to be an agent/broker of Steamship Mutual.

Although Pioneer is already licensed as an insurance company, it needs a separate


license to act as insurance agent for Steamship Mutual. Section 299 of the Insurance
Code clearly states:

SEC. 299 . . .

No person shall act as an insurance agent or as an insurance broker in the solicitation


or procurement of applications for insurance, or receive for services in obtaining
insurance, any commission or other compensation from any insurance company doing
business in the Philippines or any agent thereof, without first procuring a license so to
act from the Commissioner, which must be renewed annually on the first day of
January, or within six months thereafter. . .

Finally, White Gold seeks revocation of Pioneer’s certificate of authority and removal of
its directors and officers. Regrettably, we are not the forum for these issues.

 
Philamlife v Ansaldo, GR 76452, 26 July 1994- Benito

PHILIPPINE AMERICAN LIFE INSURANCE COMPANY vs. HON. ARMANDO ANSALDO

G.R. No. 76543 | July 26, 1994

Topic: Office of the Insurance Commission. Admin and Adjudicatory Powers

Quiason, J.

FACTS:

·         Private respondent Paterno wrote a letter-complaint to commissioner Ansaldo alleging


certain problems encountered by agents and consumers of Philamlife, particularly on the
illegality of the Contract of Agency (CoA).

·         A hearing was held on the validity of the said CoA, wherein Paterno prayed that the
provisions on charges and fees stated in the CoA executed between Philamlife and its
agents, as well as the implementing provisions, be declared null and void.

o   He also prayed to reimburse agents of the amounts deducted from charges and
fees already collected with interest.

·         Philamlife contests that since the Insurance Commission’s quasi-judicial power was
being invoked, Paterno must first file a verified formal complaint before any further
proceedings.

·         Philamlife also filed a Motion to Quash Subpoena on the ground that the Commission
has no jurisdiction over the subject matter of the action and the parties involved. This Motion
was denied, hence this petition.

ISSUE: W/N the resolution of the illegality of the Contract of Agency falls within the jurisdiction
of the Insurance Commissioner

HELD: No, the Commissioner has no authority because the CoA entered into between
Philamlife and its agents is not included within the meaning of an insurance business under
Sec. 2(2) of the insurance Code. Expressio unius est exclusio alterius.

PETITION GRANTED. Order of Insurance Commissioner is SET ASIDE.

RATIO:

·         Under Sec. 414 and 415 of the Insurance Code, the Commissioner has authority to
regulate the business of insurance, defined under Sec. 2(2). This “business” doesn’t include
the CoA entered into by Philamlife and its agents.

·         Neither can the quasi-judicial power of the Commissioner under Sec. 416 be invoked.
o   A reading of Sec. 416 shows that the quasi-judicial power of the Insurance
Commissioner is limited by law "to claims and complaints involving any loss, damage
or liability for which an insurer may be answerable under any kind of policy or contract
of insurance," Hence, this power does not cover the relation affecting the insurance
company and its agents but is limited to adjudicating claims and complaints filed by the
insured against the insurance company.

o   The Insurance Code does not have provisions governing the relations between
insurance companies and their agents. While the topic of Insurance Agents and
Brokers are discussed in Chapter IV of the Code, the provisions only speak of
licensing requirements and limitations imposed on insurance agents and brokers.

o   It follows that the Insurance Commissioner cannot, in the exercise of its quasi-
judicial powers, assume jurisdiction over controversies between the insurance
companies and their agents.

·         Disputes involving insurance companies and agents, such as in this case, may be
cognizable by regular courts. 

FOR JUNE 28 SESSION - 9 cases


G.R. No. L-2294             May 25, 1951
FILIPINAS COMPAÑIA DE SEGUROS, petitioner,
vs.
CHRISTERN, HUENEFELD and CO., INC., respondent.
On October 1, 1941, the respondent corporation, Christern Huenefeld, & Co., Inc., after payment of
corresponding premium, obtained from the petitioner ,Filipinas Cia. de Seguros, fire policy.
On February 27, 1942, or during the Japanese military occupation, the building and insured
merchandise were burned. In due time the respondent submitted to the petitioner its claim under the
policy.
The petitioner ( de SEGUROS) refused to pay the claim on the ground that the policy in favor of the
respondent had ceased to be in force on the date the United States declared war against Germany,
the respondent Corporation (though organized under and by virtue of the laws of the Philippines)
being controlled by the German subjects and the petitioner being a company under American
jurisdiction when said policy was issued on October 1, 1941.
The petitioner, however, in pursuance of the order of the Director of Bureau of Financing, Philippine
Executive Commission, dated April 9, 1943, paid to the respondent the sum of P92,650 on April 19,
1943.
ISSUE:
WON petitioner has cause of action for the reimbursement of the proceeds of insurance initially paid
to respondent when the policy had ceased to be in force when US declared war against Germany.
Ruling:
The respondent having become an enemy corporation on December 10, 1941, the insurance
policy issued in its favor on October 1, 1941, by the petitioner (a Philippine corporation) had
ceased to be valid and enforcible, and since the insured goods were burned after December
10, 1941, and during the war, the respondent was not entitled to any indemnity under said
policy from the petitioner. However, elementary rules of justice (in the absence of specific
provision in the Insurance Law) require that the premium paid by the respondent for the

 
CASE 28

G.R. No. L-1669             August 31, 1950


 
PAZ LOPEZ DE CONSTANTINO, plaintiff-appellant, vs.
ASIA LIFE INSURANCE COMPANY, defendant-appellee.
G.R. No. L-1670             August 31, 1950
AGUSTINA PERALTA, plaintiff-appellant,
vs.
ASIA LIFE INSURANCE COMPANY, defendant-appellee.

The facts are these:

First case.

Asia Life Insurance Company (a foreign corporation incorporated under the laws of Delaware,
U.S.A.), issued on September 27, 1941, its Policy No. 93912 for P3,000, whereby it insured the
life of Arcadio Constantino for a term of twenty years. The first premium covered the period up
to September 26, 1942. After that first payment, no further premiums were paid. The insured
died on September 22, 1944

Second case.

 On August 1, 1938, the defendant Asia Life Insurance Company issued its Policy No. 78145
(Joint Life 20-Year Endowment Participating with Accident Indemnity), covering the lives of the
spouses Tomas Ruiz and Agustina Peralta, for the sum of P3,000.  There was novation on the
terms of payment of insurance premium from annual payments to quarterly payments. After
initial performance of premium was made, No further payments were handed to the insurer.
Upon the Japanese occupation, the insured and the insurer became separated by the lines of
war, and it was impossible and illegal for them to deal with each other.

Issue:

Whether or not policies had lapsed for non-payment of premium.

Ruling:

The non-payment of premiums is such a vital defense of insurance companies that since the
very beginning. And when Congress recently amended this section (Rep. Act No. 171), the defense
of fraud was eliminated, while the defense of nonpayment of premiums was preserved. Thus the
fundamental character of the undertaking to pay premiums and the high importance of the defense
of non-payment thereof, was specifically recognized.

CASE#03 Fidela Sales Gonzaga v Crown Life, GR L-4197, 20 March 1957


FACTS:
1939, Crown Life Insurance Co., (Toronto, Canada)
> issued Ramon Gonzaga through its Manila office, a 20-year endowment policy for P15,000.
> The insured paid the yearly premium (P591.00), for three consecutive years,
> the last payment: September 6, 1941.
 
The war broke out, and no premiums were paid after that date,
> the policy was still in force up to June 12, 1943, under its automatic premium loan clause.

Ramon Gonzaga died on June 27, 1945 from an accident.


 
His widow Fidela, filed this suit on December 18, 1947,
> because she was not able to collect the proceeds from the policy
 
Crown Life argued that the policy had lapsed because of non-payment of premiums
 
RTC ruled against widow Fidela.
 
Fidela asserted that it was Crown Life’s duty to notify her husband of its postal address during the war,
and that its failure to do so, excused delinquency in the payment of the premiums.
 
Crown Life alleged that it had its offices open in the City of Manila during the Japanese occupation of the
Philippines through its General Agents, Hanson, Orth & Stevenson, Inc.,

ISSUE: W/N the widow is entitled to the benefits of her deceased husband’s insurance policy
 
HELD: NO 
The SC in previous cases held that… "time is material and of the essence of the contract. Non-payment
at the day involves absolute forfeiture, if such be the terms of the contract… and Courts cannot vary the
stipulation of the parties by introducing equities for the relief of the insured, against their own negligence."
 
Meaning: non-payment of premiums by reason of war puts an end to the contract.

SC held that Crown Life being an enemy corporation, had its offices closed by the Japanese in January
1942, and the officers it’s general agents, being American citizens, were not able to operate.
 
Moreover, the Japanese administration issued an order that enemy alien insurance companies were
expressly prohibited from doing business.

It is a fact that before the order of the Japanese was promulgated… Crown Life’s general agent had
opened in the house of one of their Filipino employees in Ermita, an office with a skeleton force-- all
Filipinos, for the purpose of receiving premiums from their policy holders…

The failure of Crown Life or its Filipino employees to advise the insured of the defendant’s new address
did not work as a forfeiture of its right to have the premiums satisfied promptly.
 
Meaning: Crown Life still had a right to forfeit the premium, even if it did not advise the policy holder of its
new address.
 
Transactions during war are binding, but it was not obligatory on the insurer because it was too risky for
the employees to send out notices to policy holders—given that the postal service was under the control
of the Japanese. There is no duty where the law forbids; and there is no obligation without a
corresponding right enjoyed by another.
 
The insured had no right to demand that the defendant maintain an office during the war, and the
defendant was not obligated to do so.
 
In any case, even if Crown Life did not open the Ermita office, Fidela’s situation would be the same. Even
if the deceased paid his premiums and was rejected, he did not have a right to insist on such payment –
since the insurer had no such duty to accept.
 
Crown Life’s opening of an office was a privilege to the policy holders so that their policies remain
operative—but it was not a duty to them by Crown Life.

Furthermore, the deceased, if he wanted to keep his policy, could have contacted his agent and the other
Filipino employees of Crown Life—who were incidentally, his close acquaintances.

AND the policy carried a clause providing for its reinstatement under certain conditions within three years
from the date of lapse…and the deceased could have applied for this.
> June 12, 1943 -- the policy lapsed
> May 1, 1945, --  Manila branch reopened and resumed regular business at the Wilson Building 
on Juan Luna Street, Manila
> June 27, 1945, Ramon Gonzaga died.
 
Ramon undoubtedly knew all that, because he was an employee in the United States Navy, WHICH had
an office in the same Wilson Building, --wherein he came at least twice a month for his salary.

Hence, the action was properly dismissed and the appealed decision is affirmed.

MORTGAGOR-MORTGAGEE

CASE#04 
Palileo v Cosio, GR L-7667, 28 November 1955
BONDOC

CASE TITLE: CHERIE PALILEO  vs. BEATRIZ COSIO


Plaintiff: CHERIE PALILEO  (mortgagor)
Defendant: BEATRIZ COSIO (mortgagee)
G.R. No. L-7667           
Date: November 28, 1955
PONENTE: BAUTISTA ANGELO, J.
Claro M. Recto for appellant.
 
FACTS:
On December 19, 1952,
defendant BEATRIZ COSIO
filed her answer setting up as special defense
that the transaction entered into between the plaintiff and defendant
is one of “sale with option to repurchase”
 
but that the period for repurchase had expired
without plaintiff CHERIE PALILEO having returned the price agreed upon
as a result of which, the ownership of the property had become consolidated in the defendant-
mortgagee.
 
(Defendant also set up certain counterclaims which involve a total amount of P4,900)
 
THE COMPLAINT
Plaintiff filed a complaint against defendant in the CFI of Manila
praying that;
 
(1)
 the transaction entered into between them on December 18, 1951
         --> be declared as one of loan: and
 
the document executed covering the transaction
         --> be declared as one of equitable mortgage to secure the payment of said loan;
 
(2)
the defendant be ordered to credit to the plaintiff with the necessary amount from the sum
received by the defendant from the Associated Insurance & Surety Co., Inc,;
and to apply the same to the payment of plaintiff-mortgagor’s obligation thus considering it as
fully paid; and
 
(3) the defendant be ordered to pay to plaintiff the difference between the alleged indebtedness
of plaintiff and the sum received by defendant from the aforementioned insurance company,
plus the sum allegedly paid to defendant as interest on the alleged indebtedness.
 
 
 
 
 
 
LOWER COURT’S FINDINGS
On December 18, 1951,
Plaintiff-mortgagor
obtained a loan from defendant
in the sum of P12,000.
The loan was subject to the following 3 conditions:
(a) interest in the amount of P250 a month;
 
(b) defendant-mortgagee shall deduct from the loan certain obligations of plaintiff to third persons
amounting to P4,550, plus the sum of P250 as interest for the first month; and
 
(c) that after making the above deductions, defendant-mortgagee shall deliver to plaintiff only the
balance of the loan of P12,000.
 
 
Pursuant to their agreement,
plaintiff paid to defendant as interest on the loan a total of P2,250.00
corresponding to 9 months from December 18, 1951,
on the basis of P250.00 a month,
(which, by the way, is more than the maximum interest authorized by law).
 
 
 
TO SECURE THE PAYMENT OF THE AFORESAID LOAN:
Defendant required plaintiff to sign a document:
--> known as "Conditional Sale of Residential Building",
--> purporting to convey A TWO-STORY BUILDING to defendant, with right to repurchase,
 
THIS DOCUMENT DID NOT EXPRESS THE TRUE INTENTION OF THE PARTIES
WHICH WAS MERELY TO PLACE SAID PROPERTY AS SECURITY
FOR THE PAYMENT OF THE LOAN.
 
 
After the execution of the aforesaid document,
DEFENDANT- MORTGAGEE INSURED THE BUILDING AGAINST FIRE
with the Associated Insurance & Surety Co., Inc.
for the sum of P15,000,
the insurance policy having been issued in the name of defendant-mortgagee
 
THE BUILDING WAS PARTLY DESTROYED BY FIRE
and, after proper demand,
defendant-mortgagee collected from the insurance company
an indemnity of P13,107.00.
 
PLAINTIFF-MORTGAGOR
demanded from defendant-mortgagee
THAT SHE BE CREDITED WITH THE NECESSARY AMOUNT TO PAY HER OBLIGATION OUT
OF THE INSURANCE PROCEEDS
 
BUT DEFENDANT REFUSED TO DO SO.
 
TRIAL COURT’S DECISION: it was an EQUITABLE MORTGAGE
 
--> declared the transaction had between plaintiff and defendant as AN EQUITABLE MORTGAGE
to secure the payment of the sum of P12,000 loaned by the defendant to plaintiff;
--> TRIAL COURT ORDERED THE DEFENDANT-MORTGAGEE TO CREDIT THE SUM OF
P13,107 RECEIVED BY THE DEFENDANT(from Insurance) TO THE PAYMENT OF PLAINTIFF'S
OBLIGATION IN THE SUM OF P12,000.00
--> Thus, due to this, the agreement of December 18, 1951 (loan) between the herein plaintiff
and defendant will be deemed COMPLETELY PAID
--> and leaving still a balance in the sum of P1,107 from the insurance collected by defendant.
 
--> this Court hereby likewise orders the said defendant to refund
to Plaintiff the overpaid amount
         --> plus the balance of P1,107
         --> representing the difference of the sum loan of P12,000 and the collected insurance of
P13,107 from the insurance company
abovementioned to which the herein plaintiff is entitled to receive
 
 
ISSUE:
 
Is the trial court justified in considering the obligation of plaintiff fully compensated
by the insurance amount
and in ordering defendant-mortgagee to refund to plaintiff
the sum of P1,107
<representing the difference of the loan of P12,000 and the sum of P13,107>
collected by said defendant from the insurance company
 
notwithstanding the fact that it was not proven that the insurance was taken for the benefit of the
mortgagor?
 
 
HELD:
 
THE COURT IS IN ERROR
FOR ITS RULING RUNS COUNTER TO THE RULE GOVERNING AN INSURANCE TAKEN BY A
MORTGAGEE INDEPENDENTLY OF THE MORTGAGOR.
 
THE RULE:
The rule is that
"where a mortgagee, independently of the mortgagor, insures the mortgaged property in his
own name and for his own interest,
 
he is entitled to the insurance proceeds in case of loss, but in such case, he is not allowed to
retain his claim against the mortgagor, 
 
but is passed by subrogation to the insurer to the extent of the money paid."
(Vance on Insurance, 2d ed., p. 654)
 
Or, stated in another way,
 
THE MORTGAGEE MAY INSURE HIS INTEREST IN THE PROPERTY
INDEPENDENTLY OF THE MORTGAGOR.
 
In that event, upon the destruction of the property,
the insurance money paid to the mortgagee will not inure to the benefit of the mortgagor, 
and the amount due under the mortgage debt remains unchanged. 
 
The mortgagee, however, is not allowed to retain his claim against the mortgagor,
 but it passes by subrogation to the insurer, to the extent of the insurance money paid.
 
 
This is the same rule upheld by this Court in a case that arose in this jurisdiction.
In a previous case,
an insurance contract was taken out by the mortgagee upon his own interest,
it being stipulated that the proceeds would be paid to him only.
And when the case came up for decision,
this Court held that the mortgagee, in case of loss, may only recover upon the policy to the
extent of his credit at the time of the loss.
It was declared that the mortgaged (mortgagor?) had no right of action against the mortgagee
on the policy.
(San Miguel Brewery vs. Law Union, 40 Phil., 674.)
 
 
THE MINORITY ViEW (which is not applicable to this case):
There are a few authorities which hold that:
"If a mortgagee procures insurance on his separate interest at his own expense and for his own
benefit, without any agreement with the mortgagor with respect thereto, the mortgagor has no
interest in the policy, and is not entitled to have the insurance proceeds applied in reduction of the
mortgage debt"
and that, furthermore, the mortgagee
"has still a right to recover his whole debt of the mortgagor."
But these authorities merely represent the minority view”
 
THE MAJORITY / APPLICABLE VIEW
The general rule and the weight of authority is,
THAT THE INSURER IS THEREUPON SUBROGATED TO THE RIGHTS OF THE MORTGAGEE
UNDER THE MORTGAGE.
THIS IS PUT UPON THE ANALOGY OF THE SITUATION OF THE INSURER TO THAT OF A
SURETY."
 
 
In the light of the majority rule,
THE CORRECT SOLUTION SHOULD BE
(1) that the proceeds of the insurance should be delivered to the defendant-mortgagee
(2) but that her claim against the plaintiff-mortgagor-debtor
should be CONSIDERED ASSIGNED to the INSURANCE COMPANY
(who is deemed subrogated to the rights of the defendant to the extent of the money paid as
indemnity.)
 
 
THUS:
(1) the transaction had between the plaintiff and defendant as shown in Exhibit A
is merely an equitable mortgage intended to secure the payment of the loan of P12,000;
 
(2) that the proceeds of the insurance amounting to P13,107.00 was properly collected by defendant
who is not required to account for it to the plaintiff;
 
(3) that the collection of said insurance proceeds shall not be deemed to have compensated
the obligation of the plaintiff to the defendant,
but bars the latter from claiming its payment from the former; and
 
(4) defendant-mortgagee shall pay to the plaintiff-mortgagor the sum of P810.00 representing the
overpayment made by plaintiff by way of interest on the loan.
 

CASE#05 San Miguel Brewery v Law Union, GR L-14300, 19 January 1920

Facts:

On Jan. 12, 1918, Dunn mortgaged a parcel of land to SMB to secure a


debt.
Mortgage contract stated that Dunn was to have the property insured at his
own expense, authorizing SMB to choose the insurers and to receive the
proceeds thereof and retain so much of the proceeds as would cover the
mortgage debt.
Dunn likewise authorized SMB to take out the insurance policy for him.
Brias, SMB’s general manager, approached Law Union for insurance to the
extent of 15T upon the property.  In the application, Brias stated that SMB’s
interest in the property was merely that of a mortgagee.
The Law Union, not wanting to issue a policy for the entire amount, issued
one for P7,500 and procured another policy of equal amount from Filipinas
Cia de Seguros.  Both policies were issued in the name of SMB only and
contained no reference to any other interests in the property. Both policies
required assignments to be approved and noted on the policy.
Premiums were paid by SMB and charged to Dunn. A year later, the
policies were renewed.
In 1917, Dunn sold the property to Harding, but no assignment of the
policies was made to the latter.
Property was destroyed by fire.  SMB filed an action in court to recover the
policies.  Harding was made a defendant because by virtue of the sale, he
became the owner of the property, although the policies were issued in
SMB’s name.
SMB sought to recover the proceeds to the extent of its mortgage credit
with the balance to go to Harding.
Insurance Companies contended that they were not liable to Harding
because their liability under the policies was limited to the insurable
interests of SMB only.
SMB eventually reached a settlement with the insurance companies and
was paid the balance of it’s mortgage credit.  Harding was left to fend for
himself.  Trial court ruled against Harding.  Hence the appeal.

Issues:
W/N San Miguel has insurable interest as mortgagor only to the extent of
the mortgage credit

Ruling:
YES. 
Section 19 of the Insurance Act:
a change of interest in any part of a thing insured unaccompanied by a
corresponding change of interest in the insurance, suspends the insurance
to an equivalent extent, until the interest in the thing and the interest in the
insurance are vested in the same person.
Section 55: the mere transfer of a thing insured does not transfer the policy,
but suspends it until the same person becomes the owner of both the policy
and the thing insured.
 
With respect to Harding, when he acquired the property, no change or
assignment of the policies had been undertaken.  The policies might have
been worded differently so as to protect the owner, but this was not done.

CASE#06 Gonzalez LaO v Yek Tong Lin Fire, GR 33131, 13 Dec. 1930

Facts: 
Gonzales was issued 2 fire insurance policies by Yek for 100k covering his leaf
tobacco products.
They were stored in Gonzales’ building on Soler St., which on Jan. 11, 1928,
burned down.
Art. 3 of the Insurance policies provided that: “Any insurance in force upon all
or part of the things insured must be declared in writing by the insured and he
(insured) should cause the company to insert or mention it in the policy.
Without such requisite, such policy will be regarded as null and void and the
insured will be deprived of all rights of indemnity in case of loss.”
Notwithstanding said provision, Gonzales entered into other insurance
contracts.  When he sought to claim from Yek after the fire, the latter denied
any liability on the ground of violation of Art. 3 of the said policies.
Gonzales however proved that the insurer knew of the other insurance policies
obtained by him long before the fire, and the insurer did NOT rescind the
insurance policies in question but demanded and collected from the insured
the premiums.
 
Issue:
Whether or not Yek is still entitled to annul the contract.
 
Held:
NO.
The action by the insurance company of taking the premiums of the insured
notwithstanding knowledge of violations of the provisions of the policies
amounted to waiver of the right to annul the contract of insurance.

CASE#07 Geagonia v. CA, GR 114427, 6 February 1995 - CANADA

FACTS:
 Petitioner obtained insurance from private respondent
 It must be noted that the petitioner is also a mortgagor in this case and that he also paid
for another policy with the mortgagee as the beneficiary 
 Fire of accidental origin broke out completely destroying the insured goods
 Petitioner filed a claim against under the policy however, it was denied by the private
respondent on the ground that petitioner violated a provision regarding multiple active
insurance policies, to wit:

o “3. The insured shall give notice to the Company of any insurance or insurances
already affected, or which may subsequently be effected, covering any of the
property or properties consisting of stocks in trade, goods in process and/or
inventories only hereby insured, and unless such notice be given and the particulars
of such insurance or insurances be stated therein or endorsed in this policy pursuant
to Section 50 of the Insurance Code, by or on behalf of the Company before the
occurrence of any loss or damage, all benefits under this policy shall be deemed
forfeited, provided however, that this condition shall not apply when the total
insurance or insurances in force at the time of the loss or damage is not more than
P200,000.00.”

 Petitioner filed a complaint with the Insurance Commission which decided in favor of
petitioner and ordered the private respondent to pay the petitioner
 Private respondent elevated the case to the Court of Appeals which set aside the decision of
the Insurance Commission
 Petitioner filed this instant petition

ISSUE:
Whether or not Petitioner can claim from the Private Respondent

HELD:
YES. The petitioner can claim from the Private Respondent.

The private respondent’s denial of claims on the ground of alleged violation of the provision
regarding double insurance is not justifiable.
The Supreme Court held that the said provision’s intent is to prevent and/or discourage over-
insurance; however, in this case, there was no such thing. 

The Court stated that the insurable interests of a mortgagor and a mortgagee on the
mortgaged property are separate and distinct.

CASE#08
Saura Import & Export v Phil lnt'I Surety, GR L-15184, 31 May 1963

Facts:
On Dec. 26, 1952, Saura mortgaged to PNB its registered parcel of land in Davao to secure the
payment of a promissory note of P27T.

A building of strong materials which was also owned by Saura, was erected on the parcel of
land and the building  had always been covered by insurance even before the execution of the
mortgage contract.

Pursuant to the mortgage agreement which required Saura to insure the building and its
contents, it obtained a fire insurance for P29T from PISC for a period of 1 year starting Oct. 2,
1954.
 
The mortgage also required Saura to endorse the insurance policy to PNB.  The memo stated:
Loss if any, payable to PNG as their interest may appear, subject to the terms, conditions and
warranties of this policy.

The policy was delivered to PNB by Saura.

On Oct. 15, 1954, barely 13 days after the issuance of the fire insurance, PISC canceled the
same, effective as of the date of issue.  Notice of the cancellation was sent to PNB in writing
and was received by the bank on Nov. 8, 1954.

On Apr. 6, 1955, the building and its contents worth P4,685 were burned.  On April 11, 1985,
Saura filed a claim with PISC and mortgagee bank.

Upon presentation of notice of loss with PNB, Saura learned for the first time that the policy had
been previously canceled by PISC, when Saura’s folder in the bank’s file was opened and the
notice of the cancellation by PISC was found.

Issue:
Whether or not there was proper cancellation of the policy?

Held:
NO.
The policy in question does NOT provide for the notice of cancellation, its form or period.  The
Insurance Law does not likewise provide for such notice.  This being the case, it devolves upon
the Court to apply the generally accepted principles of insurance, regarding cancellation of the
insurance policy by the insurer. 
Actual notice of cancellation in a clear and unequivocal manner, preferably in writing should be
given by the insurer to the insured so that the latter might be given an opportunity to obtain
other insurance for his own protection.  The notice should be personal to the insurer and not to
and/or through any unauthorized person by the policy.  Both the PSIC and the PNB failed,
wittingly or unwittingly to notify Saura of the cancellation made.

The insurer contends that it gave notice to PNB as mortgagee of the property and that was
already substantial compliance with its duty to notify the insured of the cancellation of the
policy.  But notice to the bank, as far as Saura herein is concerned, is not effective notice.  PISC
is then ordered to pay Saura P29T, the amount involved in the policy subject matter of this case.

CASE#09 Phil. National Bank v CA, GR L-57757, 31 August 1987- YAP

FACTS: 

On November 28, 1952, Donata Montemayor, through her son, Salvador M. Vitug, mortgaged to the
Philippine National Bank (PNB) several parcels of land covered by Transfer Certificate of Title (TCT)
No. 2289 — Pampanga to guarantee the loan granted by the PNB to Salvador Jaramilla and Pedro
Bacani in the amount of P40,900.00 which was duly registered in the Office of the Register of Deeds
of Pampanga. 

On December 1, 1963, Donata Montemayor also mortgaged in favor of PNB certain properties
covered by TCT Nos. 2887 and 2888-Pampanga to guarantee the payment of the loan account of
her son Salvador Vitug in the amount of P35,200.00, which mortgage was duly registered in the
Register of Deeds of Pampanga. 

The above-mentioned Transfer Certificates of Titles covering said properties were all in the name of
Donata Montemayor, of legal age, Filipino, widow and a resident of Lubao, Pampanga at the time
they were mortgaged to PNB and were free from all hens and encumbrances. 

Salvador Vitug failed to pay his account so the bank foreclosed the mortgaged properties covered by
TCT Nos. 2887 and 2888. They were sold at public auction on May 20, 1968 in which the PNB was
the highest bidder. The titles thereto were thereafter consolidated in the name of PNB.
Likewise, Salvador Jaramilla and Pedro Bacani failed to settle their accounts with the PNB so the
latter foreclosed the properties covered by TCT No. 2889 which were sold at public auction and
likewise PNB was the buyer thereof. On August 30, 1968, a certificate of sale was issued by the
Register of Deeds covering said properties in favor of the PNB. When the title of the PNB was
consolidated a new title was issued in its name. 
On September 2, 1969, the PNB sold the properties covered by TCT Nos. 2887 and 2888 —
Pampanga to Jesus M. Vitug, Anunciacion V. de Guzman, Prudencia V. Fajardo, Salvador Vitug and
Aurora V. Gutierrez in those names the corresponding titles were issued. 
Clodualdo Vitug died intestate on May 20, 1929 so his estate was settled and distributed in Special
Proceeding No. 422 in the Court of First Instance of Pampanga wherein Donata Montemayor was
the Administratrix. 
Meanwhile, on May 12,1958, Donata Montemayor executed a contract of lease of Lot No. 24, which
is covered by TCT No. 2887-R in favor of her children Pragmacio and Maximo both surnamed Vitug. 
On March 21, 1970 Pragmacio Vitug and Maximo Vitug filed an action for partition and
reconveyance with damages in the Court of First Instance of Pampanga against Marcelo Mendiola,
special administrator of the intestate estate of Donata Montemayor who died earlier, Jesus Vitug,
Sr., Salvador, Natalia, Prudencia, Anunciacion, all surnamed Vitug, Antonio, Francisco, Aurora,
Pedro, Honorio, Corazon, Anselmo, Benigno, Eligio Jesus and Luz, all surnamed Fajardo and the
PNB.
The subject of the action is 30 parcels of land which they claim to be the conjugal property of the
spouses Donata Montemayor and Clodualdo Vitug of which they claim a share of 2/11 of 1/2 thereof.
They assailed the mortgage to the PNB and the public auction of the properties as null and void.
They invoked the case of Vitug vs. Montemayor, L-5297 decided by this Court on Oct. 20, 1953
which is an action for partition and liquidation of the said 30 parcels of land wherein the properties
were found to be conjugal in nature.
ISSUES:

1. THE RESPONDENT COURT OF APPEALS ERRED IN NOT RECOGNIZING THE


CONCLUSIVENESS OF THE CERTIFICATE, OF TITLE, AS PROVIDED IN ACT 496, AS
AMENDED (THE LAND REGISTRATION).
2. THE RESPONDENT COURT OF APPEALS ERRED IN CONCLUDING THAT PNB WAS A
MORTGAGEE IN BAD FAITH.

RULING:
No. The court is correct that PNB is a mortgagee in good faith and that the certificate is a conclusive
proof of ownership. 
RATIO:
When the subject properties were mortgaged to the PNB they were registered in the name of Donata
Montemayor, widow. Relying on the torrens certificate of title covering said properties the mortgage
loan applications of Donata were granted by the PNB and the mortgages were duly constituted and
registered in the office of the Register of Deeds.
In processing the loan applications of Donata Montemayor, the PNB had the right to rely on what
appears in the certificates of title and no more. On its face the properties are owned by Donata
Montemayor, a widow. The PNB had no reason to doubt nor question the status of said registered
owner and her ownership thereof. Indeed, there are no liens and encumbrances covering the same.
The well-known rule in this jurisdiction is that a person dealing with a registered land has a right to
rely upon the face of the torrens certificate of title and to dispense with the need of inquiring further,
except when the party concerned has actual knowledge of facts and circumstances that would impel
a reasonably cautious man make such inquiry. 9

A torrens title concludes all controversy over ownership of the land covered by a final degree
of The presumption applies to property acquired during the lifetime of the husband and wife.

In this case, it appears on the face of the title that the properties were acquired by Donata
Montemayor when she was already a widow. When the property is registered in the name of a
spouse only and there is no showing as to when the property was acquired by said spouse,
this is an indication that the property belongs exclusively to said spouse. And this
presumption under Article 160 of the Civil Code cannot prevail when the title is in the name of
only one spouse and the rights of innocent third parties are involved. 

The PNB had a reason to rely on what appears on the certificates of title of the properties
mortgaged. For all legal purposes, the PNB is a mortgagee in goodfaith for at the time the
mortgages covering said properties were constituted the PNB was not aware to any flaw of
the title of the mortgagor. 

True it is that in the earlier cases decided by this Court, namely Vitug VS. Montemayor decided on
May 15, 1952, which is an action for recovery of possession of a share in said parcels of land, 15
and in the subsequent action for partition between the same parties decided on Oct. 20, 1953, 16
this court found the 30 parcels of land in question to be conjugal in nature and awarded the
corresponding share to the property of Florencia Vitug, an heir of the late Clodualdo Vitug from the
first marriage. In said cases this Court affirmed the decision of the lower court. In the dispositive part
of the decision of the trial court it made the observation that "but from the conduct of Clodualdo Vitug
and Donata Montemayor during the existence of their marital life, the inference is clear that
Clodualdo had the unequivocal intention of transmitting the full ownership of the 30 parcels of land to
his wife Donata Montemayor, thus considering the 1/2 of the funds of the conjugal property so
advanced for the purchase of said parcels of land as reimbursible to the estate of Clodualdo Vitug on
his death. 17 That must be the reason why the property was registered in the name of Donata
Montemayor as widow after the death of Clodualdo Vitug. 

Lalican v. Insular Life, GR 183526, 25 August 2009

El Oriente v Posadas, GR 34774, 21 September 1931 - pascual

Facts
EL ORIENTE, FABRICA DE TABACOS, INC, plaintiff, in order to protect itself
against the loss by reason of the death of its manager, A. Velhagen, who had more
than 35 years of experience in the manufacture of cigars procured from the
Manufacturers Life Insurance Co., of Toronto, Canada, thru its local agent E. E.
Elser, an insurance policy on the life of the said A. Velhagen for the sum of
$50,000.

plaintiff, El Oriente, Fabrica de Tabacos, Inc., designated itself as the sole


beneficiary of said policy on the life of its said manager.

plaintiff paid from its funds all the insurance premiums due thereon and deducted
the same from its gross incomes as reported in its annual income tax returns, which
deductions were allowed by the defendant.

upon the death of A. Velhagen, the plaintiff received all the proceeds of life
insurance

defendant Collector of Internal Revenue assessed and levied the sum of P3,148.74
as income tax on the proceeds of the insurance policy.  

Plaintiff, paid under protest, and claimed exemption under section 4 of the Income
Tax Law, which provides that, "The following incomes shall be exempt from the
provisions of this law: (et) The proceeds of life insurance policies paid to
beneficiaries upon the death of the insured.

Trial court held that section 4 of the Income Tax Law (Act No. 2833) is not
applicable to the present case.

Thus the petitioner appealed that trial court erred in assuming that the proceeds of
the life insurance policy in question represented a net profit to the plaintiff when,
as a matter of fact, it merely represented an indemnity for the loss suffered by it
thru the death of its manager, the insured. The trial court erred in refusing to hold
that the proceeds of the life insurance policy in question is not taxable income, and
in absolving the defendant from the complaint.

Issues

whether the proceeds of insurance taken by a corporation on the life of an


important official to indemnify it against loss in case of his death, are taxable as
income under the Philippine Income Tax Law
 

Ruling

No

The proceeds of insurance taken by a corporation on the life of an important


official to indemnify it against loss in case of his death, are not taxable as income
under the Philippine Income Tax Law. The indefiniteness of the local law is
emphasized.

. It is certain that the proceeds of life insurance policies paid to individual


beneficiaries upon the death of the insured are exempt

In reference to laws of United States Income Tax Law it is clear, that when specific
reference was made to the exemptions in favor of individuals, the same is made
applicable to corporations as beneficiaries

We do not believe that when the plaintiff received P104,957.88 from the insurance
on the life of its manager, it thereby realized a net profit in this amount. It is true
that the Income Tax Law, in exempting individual beneficiaries, speaks of the
proceeds of life insurance policies as income, but this is a very slight indication of
legislative intention. In reality, what the plaintiff received was in the nature of an
indemnity for the loss which it actually suffered because of the death of its
manager

proceeds of a life insurance policy paid on the death of the insured are not usually
classed as income. "* * * Life insurance in such a case is like that of fire and
marine insurance,—a contract of indemnity. The benefit to be gained by death has
no periodicity. It is a substitution of money value for something permanently lost,
either in a house, a ship, or a life

considering the lack of express legislative intention to tax the proceeds of life
insurance policies paid to corporate beneficiaries, particularly when in the
exemption in favor of individual beneficiaries in the chapter on this subject, the
clause is inserted "exempt from the provisions of this law," we deem it reasonable
to hold the proceeds of the life insurance policy in question as representing an
indemnity and not taxable income.

Insular Life v Ebrado, GR L-44059, 28 October 1977 - pascual


Facts

On September 1, 1968, Buenaventura Cristor Ebrado was issued by The Insular


Life Assurance Co., Ltd., Policy No. 009929 on a whole-life plan for P5,882.00
with a rider for Accidental Death Benefits for the same amount. Buenaventura C.
Ebrado designated Carponia T. Ebrado as the revocable beneficiary in his policy.
He referred to her as his wife. During the lifetime of Buenaventura Ebrado, he was
living with his common-law wife, Carponia, although he was not legally separated
from his legal wife.

Carponia Ebrado and the insured made reservation to change the beneficiary but
although the insured made the option to change the beneficiary, same was never
changed up to the time of his death and the legal wife did not have any opportunity
to write the company that there was reservation to change the designation of the
beneficiary

On October 21, 1969, Buenaventura C. Ebrado died as a result of an accident when


hit by a falling tree branch

Carponia T. Ebrado filed with the insurer a claim for the proceeds of the policy as
the designated beneficiary therein, although she admits that they were common law
spouses. Thus Pascuala Vda. de Ebrado filed her claim as the widow of the
deceased insured.

Insular life filed at CFI as to whom the proceeds of the policy P11,745.73 shall be
paid. CFI declared Carponia T. Ebrado disqualified from becoming beneficiary.
CFI explained that the  last paragraph of Art. 739, NCC of criminal conviction for
adultery or concubinage is not essential in order to establish the disqualification of
Carponia. Neither it is necessary to file a separate criminal action. The guilt of the
donee (beneficiary) may be proved by preponderance of evidence in the same
proceeding (the action brought to declare the nullity of the donation).

It is, however, essential that such adultery or concubinage exists at the time
defendant Carponia T. Ebrado was made beneficiary in the policy for the
disqualification to exist. This was affirmed by CA.

Issues

Whether a common law wife named as beneficiary in the life insurance policy of a
legally married man claim the proceeds thereof in case of his death
 

Ruling

No.

proceeds of the policy are payable to the estate of the deceased insure not to the
common law wife.

Section 50 of the Insurance Act which provides that “(t)he insurance shall be
applied exclusively to the proper interest of the person in whose name it is made”1
cannot be validly seized upon to hold that the same includes the beneficiary. The
word “interest” highly suggests that the provision refers only to the “insured” and
not to the beneficiary, since a contract of insurance is personal in character

Article 2011 of the New Civil Code states: “The contract of insurance is
governed by special laws. Matters not expressly provided for in such special
laws shall be regulated by this Code.” When not otherwise specifically provided
for by the Insurance Law, the contract of life insurance is governed by the general
rules of the civil law regulating contracts. 3 And under Article 2012 of the same
Code, “any person who is forbidden from receiving any donation under
Article 739 cannot be named beneficiary of a life insurance policy by the
person who cannot make a donation to him.”4 Common-law spouses are,
definitely, barred from receiving donations from each other. Article 739 of the new
Civil Code provides:

“The following donations shall be void: “1. Those made between persons who
were guilty of adultery or concubinage at the time of donation;… the action
for declaration of nullity may be brought by the spouse of the donor or donee;
and the guilt of the donee may be proved by preponderance of evidence in the
same action.” 

the law plainly states that the guilt of the party may be proved “in the same action”
for declaration of nullity of donation. And, it would be sufficient if evidence
preponderates upon the guilt of the consort for the offense indicated. The quantum
of proof in criminal cases is not demanded.

In essence, a life insurance policy is no different from a civil donation insofar as


the beneficiary is concerned. Both are founded upon the same consideration:
liberality. A beneficiary is like a donee, because from the premiums of the policy
which the insured pays out of liberality, the beneficiary will receive the proceeds
or profits of said insurance. As a consequence, the proscription in Article 739 of
the new Civil Code should equally operate in life insurance contracts.

The mandate of Article 2012: any person who cannot receive a donation cannot be
named as beneficiary in the life insurance policy of the person who cannot make
the donation.

a beneficiary in a life insurance policy is no different from a donee. Both are


recipients of pure beneficence. So long as marriage remains the threshold of our
family law, reason and morality alike demand that the disabilities attached to
marriage should likewise attach to concubinage.

Herein, the stipulations at CFI are judicial admissions which, as a consequence, no


longer require proof and cannot be contradicted.8 A fortiori, on the basis of these
admissions, a judgment may be validly rendered without going through the rigors
of a trial for the sole purpose of proving the illicit liaison between the insured and
the beneficiary.

Vda. De Consuegra v GSIS, GR L-28093, 30 January 1971

FACTS:

This is a brief discussion about the case of Basilia viuda de Consuegra Vs


GSIS.

This case involves a claim of two diff spouses to the retirement insurance of
the late Jose Consuegra namely (Rosario Diaz and Basilia berdin Consuegra).

Jose Consuegra was first married to Rosario Diaz, but later contracted
another marriage to Basilia Berdin (Which was contracted in good faith)

Jose Consuegra died, and as a member of GSIS, his widow is entitled to the
proceeds of his retirement insurance.

(Take note that Retirement insurance and life insurance are two different
things in this case.)

The proceeds of the life insurance of Jose Consuegra were paid by GSIS to
Basilia Berdin and her children who were the beneficiaries named in the
policy.
The conflict started when they found out that consuegra was entitled to
retirement insurance and that Jose Consuegra did not designate any
beneficiary to receive such proceeds in case of death.

The widow by the first marriage Rosario Diaz filed a claim with the GSIS
asking that the retirement insurance benefits be paid to her as she is the only
legal heir of the deceased and considering that Jose Consuegra did not
designate any beneficiaries to such policy.

On the other hand the second widow Basilia Berdin contends that being the
beneficiary of Jose Consuegra’s life insurance policy. They are the ones who
are entitled to receive the retirement insurance due to the deceased.

GSIS resolves the conflict by dividing the proceeds equally to the widows, as
they are both entitled to the

The issue was elevated to the rtc and eventually up to the supreme court.

ISSUE:

To whom should the retirement insurance benefits be paid?

HELD:

Both families are entitled to half of the retirement benefits.

The contention of the second wife, that they are the sole beneficiary of the
retirement benefits through the fact that they are the named beneficiary in the
life insurance is untenable.

The provisions on retirement insurance came about only when CA 186 was
amended by RA 660 on june 1951, and the designation of beneficiaries was
made by Jose Casuegra on or before 1943.

Therefore, the beneficiaries named in the life insurance shall not


automatically become the beneficiary in the retirement insurance.

And also, although the 2 marriage can be presumed to be void ab initio.


nd

Because it was celebrated in coexistence with the first marriage.

The court cited the jurisprudence of “lao & lao vs Dee Tim, estrella vs Laong Masa
and Pisalbon vs Bejec. In which provides that, When two women innocently and
in good faith are legally united in holy matrimony to the same man, they and
their children born of said wedlock will be regarded as legitimate children
and each family will be entitled to one-half of the estate.

Therefore it is immaterial whether the second marriage is void, as long as the


woman was innocently married in good faith.

Southern Luzon Employees v Golpeo, GR L-6114, 30 October 1954


Facts:
Plaintiff, Southern Luzon Employees' Association, is composed of laborers and
employees with respect to transportation companies. One of its purposes is
mutual aid of its members and their dependents in case of death.
The association adopted the following resolution: 

xxx. “...that a member may, if he chooses, put down


his common-law wife as his beneficiary and/or children had with her as the case
may be.” xxx 

xxx. That such person so named by the member will be the sole persons to be
recognized by the Association regarding claims for condolence contributions."
xxx
 Roman A. Concepcion was a member until his death on December 13, 1950. After
the death of Roman A. Concepcion, the association was able to collect voluntary
contributions from its members amounting to P2,505. Three sets of claimants
presented themselves, namely, (1) Juanita Golpeo, legal wife of Roman A.
Concepcion, and her children; (2) Aquilina Maloles, common law wife of Roman
A. Concepcion, and her children, named beneficiaries by the deceased; and (3)
Elsie Hicban, another common law wife of Roman A. Concepcion, and her child.
The plaintiff association was accordingly constrained to institute in the Court of
First Instance of Laguna the present action for interpleading against the three
conflicting claimants as defendants. Marcelino and Josefina Concepcion, children
of the deceased Roman A. Concepcion with Juanita Golpeo, intervened in their
own rights, aligning themselves with the defendants, Juanita Golpeo and her minor
children. After hearing, the court rendered a decision, declaring the defendants
Aquilina Maloles and her children the sole beneficiaries of the sum of P2,505.00,
and ordering the plaintiff to deliver said amount to them. From this decision only
the defendants Juanita Golpeo and her minor children and the intervenors
Marcelino and Josefina Concepcion have appealed to the Supreme Court.
Issue/s: 
1. Whether or not the contract between the association and its member, Roman
Concepcion, was a contract of insurance.
2. Whether or not Aquilina and her children can be validly made as Roman’s
beneficiaries.
Held:
1. No, Southern Luzon Employees Association was not a regular insurance
company but the death benefit in question is analogous to insurance.
Moreover, Section 1628 of the Revised Administrative Code defines a mutual
benefit association as one, among others, "providing for any method of
accident or life insurance among its members out of dues or assessments
collected from the membership.” The contract of life insurance is a special
contract and the destination of the proceeds thereof is determined by special
laws which deal exclusively with that subject. That subject is regulated
exclusively by the Code of Commerce which provides for the terms of the
contract, the relations of the parties and the destination of the proceeds of the
policy.
          2. Yes. The lower court invoked the pronouncements in the case of Del Val
vs Del Val that the agreement being analogous to a contract of insurance, then
the amount in question belong exclusively to Aquilina and her children and
not to the estate of Roman Concepcion, and that such proceeds are separate
and individual property of the beneficiary and not of the heirs of the person
insured. This doctrine is supported by Section 428 of the Code of Commerce
which provides that:
           “The amount which the underwriter must deliver to the person insured, in
fulfillment of the contract, shall be the property of the latter, even against the
claim of the legitimate heirs or creditors of any kind whatsoever of the person
who effected the insurance in favor of the former. “

           Appellants also argue that Aquillana Maloles and her children cannot avail
of the benefits due to the fact that they are the illegitimate children of Roman
Conception. However, the fact that new Civil Code recognizes certain
successional rights of illegitimate children. (Article 287) diminishes that
argument.

Note: 
The contract here involved was perfected before the new Civil Code took effect,
and hence its provisions cannot be made to apply retroactively.

The contract was perfected on September 17, 1949. 

The New Civil Code of the Philippines took effect on August 30, 1950.

Heirs of Maramag v Maramag, GR 181132, 5 June 2009

Facts:

The case stems from a petition filed against respondents for revocation and/or
reduction of insurance proceeds for being void and/or inofficious.

The petition alleged that: 

1. petitioners were the legitimate wife and children of Loreto Maramag


(Loreto), while respondents were Loreto’s illegitimate family; 

2. Eva de Guzman Maramag (Eva) was a concubine of Loreto and a suspect in


the killing of the latter, thus, she is disqualified to receive any proceeds from his
insurance policies from Insular Life and Grepalife 

3. the illegitimate children of Loreto—Odessa, Karl Brian, and Trisha Angelie


—were entitled only to one-half of the legitime of the legitimate children, thus, the
proceeds released to Odessa and those to be released to Karl Brian and Trisha
Angelie were inofficious and should be reduced; and

4. petitioners could not be deprived of their legitimes, which should be


satisfied first.

ANSWER: (INSURER)

- Insular admitted that Loreto misrepresented Eva as his legitimate wife and
Odessa, Karl Brian, and Trisha Angelie as his legitimate children, and
- that they filed their claims for the insurance proceeds of the insurance
policies; 

- that when it ascertained that Eva was not the legal wife of Loreto, it
disqualified her as a beneficiary and divided the proceeds among Odessa, Karl
Brian, and Trisha Angelie, as the remaining designated beneficiaries; 

- and that it released Odessa’s share as she was of age, but withheld the
release of the shares of minors Karl Brian and Trisha Angelie pending submission
of letters of guardianship. 

 Insular alleged that the complaint or petition failed to state a cause of action
insofar as it sought to declare as void the designation of Eva as beneficiary,
because Loreto revoked her designation and it disqualified her in Policy

Insular further claimed that it was bound to honor the insurance policies
designating the children of Loreto with Eva as beneficiaries pursuant to Section 53
of the Insurance Code.

In its own answer with compulsory counterclaim:

Grepalife alleged 

- that Eva was not designated as an insurance policy beneficiary; 

- that the claims filed by Odessa, Karl Brian, and Trisha Angelie were denied
because Loreto was ineligible for insurance due to a misrepresentation in his
application form

- that the case was premature, there being no claim filed by the legitimate
family of Loreto; and that the law on succession does not apply where the
designation of insurance beneficiaries is clear.

Issues:
Whether or not illegitimate children can be beneficiaries in an insurance contract.

Ruling:
Yes.
Section 53 of the Insurance Code states that the insurance proceeds shall be applied
exclusively to the proper interest of the person in whose name or for whose benefit it is
made unless otherwise specified in the policy.
Pursuant thereto, it is obvious that the only persons entitled to claim the insurance
proceeds are either the insured, if still alive; or the beneficiary, if the insured is already
deceased, upon the maturation of the policy.
The exception to this rule is a situation where the insurance contract was intended to
benefit third persons who are not parties to the same in the form of favorable
stipulations or indemnity.
In such a case, third parties may directly sue and claim from the insurer.
Petitioners are third parties to the insurance contracts with Insular and Grepalife and,
thus, are not entitled to the proceeds thereof.
Accordingly, respondents Insular and Grepalife have no legal obligation to turn over the
insurance proceeds to petitioners. 
The revocation of Eva as a beneficiary in one policy and her disqualification as such in
another are of no moment considering that the designation of the illegitimate children as
beneficiaries in Loreto’s insurance policies remains valid.
Because no legal proscription exists in naming as beneficiaries the children of illicit
relationships by the insured, the shares of Eva in the insurance proceeds, whether
forfeited by the court in view of the prohibition on donations under Article 739 of the Civil
Code or by the insurers themselves for reasons based on the insurance contracts, must
be awarded to the said illegitimate children, the designated beneficiaries, to the
exclusion of petitioners.
It is only in cases where the insured has not designated any beneficiary, or when the
designated beneficiary is disqualified by law to receive the proceeds, that the insurance
policy proceeds shall redound to the benefit of the estate of the insured.

Philamcare v CA, GR 125678, 18 March 2002

Facts:

 -Ernani Trinos, deceased husband of respondent Julita Trinos, applied for a health
care coverage with petitioner Philamcare Health Systems, Inc. In the standard
application form, he answered no to all the questions regarding high blood
pressure, heart trouble, diabetes, cancer, liver disease, asthma or peptic ulcer. His
application was approved for a period of one year, March 1988 to March 1989.
After termination it was subsequently renewed.

-Upon the termination of the agreement, the same was extended for another year
from March 1, 1989 to March 1, 1990, then from March 1, 1990 to June 1, 1990.
The amount of coverage was increased to a maximum sum of P75,000.00 per
disability.
-Ernani suffered a heart attack, while her husband was in the hospital, respondent
tried to claim the benefits under the health care agreement. However, petitioner
denied her claim saying that the Health Care Agreement was void. According to
petitioner, there was a concealment regarding Ernani’s medical history.
-Doctors at the MMC allegedly discovered at the time of Ernani’s confinement that
he was hypertensive, diabetic and asthmatic, contrary to his answer in the
application form. Thus, respondent paid the hospitalization expenses herself,
amounting to about P76,000.00.
-After his discharge, Ernani was in and out of the Chinese General Hospital where
he later on died.
-The respondent filed a case in the RTC for action for damages against the
petitioner and its president, Dr. Benito Reverente.
RTC: ruled in favor of Julita, and ordered the defendants to reimburse the medical
expenses and was awarded moral, and exemplary damages.
CA: affirmed RTC decision except that it deleted awards for damages and
absolved Dr. Reverente.

Issues: WON the health care agreement is not an insurance contract.

Ruling: Yes. The health care agreement was in the nature of non-life
insurance, which is primarily a contract of indemnity. The insurable interest
of respondent’s husband in obtaining the health care agreement was his
own health. Once the member incurs hospital, medical or any other
expense arising from sickness, injury or other stipulated contingent, the
health care provider must pay for the same to the extent agreed upon
under the contract.

Elements of an Insurance Contract


Section 2 (1) of the Insurance Code defines a contract of insurance as an
agreement whereby one undertakes for a consideration to indemnify
another against loss, damage or liability arising from an unknown or
contingent event. An insurance contract exists where the following
elements concur:
(a) The insured has an insurable interest;
(b) The insured is subject to a risk of loss by the happening of the
designated peril;
(c) The insurer assumes the risk;
(d) Such assumption of risk is part of a general scheme to distribute actual
losses among a large group of persons bearing a similar risk; and
(e) In consideration of the insurer’s promise, the insured pays a premium.
 

NARIO vs. PHIL AM LIFE, GR L-22796, 26 JUNE 1967 - ALMOCERA

FACTS:

Mrs. Alejandra Santos-Nario applied for and was issued a life Insurance
policy (No. 503617) by the Philippine American Life Insurance Co.,
(PHILAMLIFE) under a 20-year endowment plan, with a face value of
P5,000.00.  Her husband Delfin Nario and their unemancipated son Ernesto
Nario were her revocable beneficiaries. Mrs. Alejandra Santos-Nario then
applied for a loan on the above policy with PHILAMLIFE which she is
entitled to as policy holder, after the policy has been in force for three (3)
years. The purpose of such loan was for the school expenses of Ernesto Nario.
The application bore the written signature and consent of Delfin Nario in two
capacities: first, as one of the irrevocable beneficiaries of the policy; and the
other, as father-guardian of Ernesto Nario and also the legal administrator of
the minor’s properties pursuant to Article 320 of the Civil Code of the
Philippines. PHILAMLIFE denied the loan application contending that
written consent of the minor son must not only be given by his father as legal
guardian but it must also be authorized by the court in a competent
guardianship proceeding. Mrs. Alejandra Santos-Nario then signified her
decision to surrender her policy and demand its cash value which then
amounted to P520.00. PHILAMLIFE also denied the surrender of the policy
on the same ground as that given in disapproving the loan application. Mrs.
Alejandra Santos-Nario sued PHILAMLIFE praying that the latter grant
their loan application and/or accept the surrender of said policy in exchange
for its cash value. PHILAMLIFE contends that the loan application and the
surrender of the policy involved acts of disposition and alienation of the
property rights of the minor, said acts are not within the power of
administrator granted under article 320 in relation to article 326 of the Civil
Code, hence court authority is required.

 ISSUE:

Whether or not PHILAMLIFE was justified in refusing to grant the loan


application and the surrender of the policy.

RULING:

YES. SC agreed with the lower court that the vested interest or right of the
beneficiaries in the policy should be measured on its full face value and not on
its cash surrender value, for in case of death of the insured, said beneficiaries
are paid on the basis of its face value and in case the insured should
discontinue paying premiums, the beneficiaries may continue paying it and
are entitled to automatic extended term or paid-up insurance options, etc. and
that said vested right under the policy cannot be divisible at any given time.
SC likewise agreed with the conclusion of the lower court that the proposed
transactions in question constitute acts of disposition or alienation of property
rights and not merely of management or administration because they involve
the incurring or termination of contractual obligations. The full face value of
the policy is P5,000.00 and the minor’s vested interest therein, as one of the
two (2) irrevocable beneficiaries, consists of one-half (½) of said amount or
P2,500.00. Applying laws (CC and rules of Court), the father must file a
petition for guardianship and post a guardianship bond. In the case at bar, the
father did not file any petition for guardianship nor post a guardianship bond,
and as such cannot possibly exercise the powers vested on him as legal
administrator of the minor’s property. The consent gives for and in behalf of
the son without prior court authorization to the loan application and the
surrender was insufficient and ineffective and PHILAMLIFE was justified in
disapproving the said applications. Assuming that the property of the ward
was less than P2,000, the effect would be the same, since the parents would
only be exempted from filing a bond and judicial authorization, but their acts
as legal administrators are only limited to acts of management or
administration and not to acts of encumbrance or disposition.

G.R. No. L-54216 July 19, 1989

THE PHILIPPINE AMERICAN INSURANCE COMPANY vs.


HONORABLE GREGORIO G. PINEDA in his capacity as Judge of the
Court of First Instance of Rizal, and RODOLFO C. DIMAYUGA

FACTS

·         Private respondent procured an ordinary life insurance policy from the


petitioner company and designated his wife and children as irrevocable
beneficiaries of said policy.
·         Private respondent filed a petition before the CFI of Rizal to amend the
designation of the beneficiaries in his life policy from irrevocable to revocable.

·         Petitioner filed an Urgent Motion to Reset Hearing. Also on the same date,
petitioner filed its Comment and/or Opposition to Petition.

·         When the petition was called for hearing on, the respondent Judge Gregorio G.
Pineda, presiding Judge of the then CFI of Rizal, denied petitioner's Urgent
Motion, thus allowing the private respondent to adduce evidence, the consequence
of which was the issuance of the questioned Order granting the petition.

·         Petitioner promptly filed a Motion for Reconsideration but the same was
denied.

ISSUE:

WHETHER OR NOT THE DESIGNATION OF THE IRREVOCABLE


BENEFICIARIES COULD BE CHANGED OR AMENDED WITHOUT THE
CONSENT OF ALL THE IRREVOCABLE BENEFICIARIES.

RULING: NO

Needless to say, the applicable law in the instant case is the Insurance Act,
otherwise known as Act No. 2427 as amended, the policy having been procured in
1968. Under the said law, the beneficiary designated in a life insurance contract
cannot be changed without the consent of the beneficiary because he has a
vested interest in the policy.

POLICY AND LAW DOES NOT PROVIDES FOR EXCEPTION

Inevitably therefore, based on the aforequoted provision of the contract, not to


mention the law then applicable, it is only with the consent of all the beneficiaries
that any change or amendment in the policy concerning the irrevocable
beneficiaries may be legally and validly effected. Both the law and the policy do
not provide for any other exception, thus, abrogating the contention of the private
respondent that said designation can be amended if the Court finds a just,
reasonable ground to do so.
DEATH OF BENEFICIARY-WIFE IS NO A RATIFICATION TO THE
CHANGE OF THE BENEFICIARIES FROM IRREVOCABLE TO
REVOCABLE.

NO CONSENT OF 6 MINOR CHILDREN

The insured ... can do nothing to divest the beneficiary of his rights without
his consent. He cannot assign his policy, nor even take its cash surrender value
without the consent of the beneficiary. Neither can the insured's creditors seize the
policy or any right thereunder. The insured may not even add another beneficiary
because by doing so, he diminishes the amount which the beneficiary may recover
and this he cannot do without the beneficiary's consent.

PARENT-INSURED CANNOT EXERCISE

Therefore, the parent-insured cannot exercise rights and/or privileges pertaining to


the insurance contract, for otherwise, the vested rights of the irrevocable
beneficiaries would be rendered inconsequential.

Villanueva v Oro, GR L-2227, 31 August 1948 - DATILES

Facts
The West Coast Life Insurance Company issued two policies of insurance on the life of
Esperanza J. Villanueva, one for P2,000, maturing on April 1, 1943 and the other for P3,000,
maturing on March 31, 1943. In both policies, the insurer agreed to pay the P2,000 to either
Esperanza, if still living on April 1, 1943; or to beneficiary Bartolome Villanueva, father of the
insured, immediately upon receipt of due proof of the prior death of the insured, Esperanza J.
Villanueva, of La Paz, Philippine Islands, during the continuance of this policy, with right on the
part of the insured to change the beneficiary.
 
In 1940, Bartolome died, and Mariano Villanueva, brother of Esperanza, substituted him as
beneficiary under the policies.
 
On October 15, 1944, Esperanza died without having collected the insurance proceeds. Adverse
claims for said proceeds were presented by the estate of Esperanza Villanueva and by Mariano J.
Villanueva on the other.
 
Court of First Instance held that the estate of Esperanza was entitled to the proceeds to the
exclusion of the beneficiary, Mariano J. Villanueva, the latter has interposed the present appeal.
Issues: WON the beneficiary is entitled to the proceeds.

Ruling: NO
Under the policies, the insurer obligated itself to pay the insurance proceeds (1) to the insured if
the latter lived on the dates of maturity or (2) to the beneficiary if the insured died during the
continuance of the policies. The first contingency excludes the second, and vice versa. In other
words, as the insured Esperanza Villanueva was living on April 1 and March 31, 1943, the
proceeds are payable exclusively to her estate unless she had before her death otherwise assigned
the matured policies. It was not proven that there was such assignment in the case. The
beneficiary, Mariano Villanueva, could be entitled to the said proceeds only in default of the first
contingency. To sustain the beneficiary's claim would be altogether eliminate from the policies
the condition that the insurer “agrees to pay . . . to the insured hereunder, if living.”
 
Under the Insurance Law, it provides that “an insurance upon life may be made payable on the
death of the death of the person, or on his surviving a specified period, or otherwise, contingently
on the continuance or cessation of life” and that “a policy of insurance upon life or health mat
pass by transfer, will, or succession, to any person, whether he has an insurable interest or not,
and such person may recover upon it whatever the insured might have recovered.”
The beneficiary claims that the proceeds of an insurance policy belong exclusively to the
beneficiary and not to the estate of the person whose life was insured, and that such proceeds are
the separate and individual property of the beneficiary, and not of the heirs of the person whose
life was insured.
 
The Supreme Court claimed that such contention was not controlling because it does not appear
therein that the insurance contract contained the stipulation appearing in the policies issued on
the life of Esperanza Villanueva and on which the appealed order in the case at bar is based.
 
The interest of the insured in the proceeds of the insurance depends upon his survival of the
expiration of endowment period. Upon the insured's death, within the period, the beneficiary will
take, as against the personal representative or the assignee of the insured. Upon the other hand, if
the insured survives the endowment period, the benefits are payable to him or to his assignee,
notwithstanding a beneficiary is designated in the policy.

Harvardian College v Country Bankers (CA), CA CV 03771, Jan. 6, 1986, 1


CARA 1

Facts: 

Harvardian is a family corporation, the stockholders of which are


Ildefonso Yap, Virginia King Yap and their children.
Prior to Aug. 9, 1979, an agent of Country Bankers proposed to
Harvardian to insure its school building.  Although at first
reluctant, Harvardian agreed.
Country Banks sent an inspector to inspect the school building and
agreed to insure the same for P500,000 for which Harvardian paid
an annual premium of P2,500.
On Aug. 9, 1979, Country Bankers issued to Harvardian a fire
insurance policy.  On March 12, 1980, during the effectiveness of
said insurance policy, the insured property was totally burned
rendering it a total loss.
A claim was made by plaintiff upon defendant but defendant denied
it contending that plaintiff had no insurable interest over the
building constructed on the piece of land in the name of the late
Ildefonso Yap as owner.
It was contended that both the lot and the building were owned by
Ildefonso Yap and NOT by the Harvardian Colleges.
 

Issues:

Whether Harvardian Colleges have a right to the proceeds or not?

Ruling:

YES. Harvardian have a right to the proceeds.

Regardless of the nature of the title of the insured or even if he did


not have title to the property insured, the contract of fire insurance
should still be upheld if his interest in or his relation to the
property is such that he will be benefited in its continued existence
or suffer a direct pecuniary loss from its destruction or injury.  The
test in determining insurable interest in property is whether one
will derive pecuniary benefit or advantage from its preservation, or
will suffer pecuniary loss or damage from its destruction,
termination or injury by the happening of the event insured against.
Here, Harvardian was not only in possession of the building but
was in fact using the same for several years with the knowledge and
consent of Ildefonso Yap.  It is reasonably fair to assume that had
the building not been burned, Harvardian would have been allowed
the continued use of the same as the site of its operation as an
educational institution.  Harvardian therefore would have been
directly benefited by the preservation of the property, and certainly
suffered a pecuniary loss by its being burned.

 
Gaisano v Insurance Co. of North America, GR 147839, 8 June 2006

Facts: Intercapitol Marketing Corporation (IMC) is the maker of Wrangler Blue Jeans.
Levi Strauss (Phils.) Inc. (LSPI) is the local distributor of products bearing trademarks
owned by Levi Strauss & Co.. IMC and LSPI separately obtained from respondent fire
insurance policies with book debt endorsements. The insurance policies provide for
coverage on "book debts in connection with ready-made clothing materials which have
been sold or delivered to various customers and dealers of the Insured anywhere in the
Philippines."  The policies defined book debts as the "unpaid account still appearing in
the Book of Account of the Insured 45 days after the time of the loss covered under this
Policy."

- Petitioner is a customer and dealer of the products of IMC and LSPI. On February 25,
1991, the Gaisano Superstore Complex in Cagayan de Oro City, owned by petitioner,
was consumed by fire. Included in the items lost or destroyed in the fire were stocks of
ready-made clothing materials sold and delivered by IMC and LSPI.

-Respondent filed a complaint for damages against petitioner. It alleges that IMC and
LSPI filed with respondent their claims under their respective fire insurance policies with
book debt endorsements; the unpaid accounts of petitioner on the sale and delivery of
ready-made clothing materials with IMC.

-respondent was subrogated to their rights against petitioner; that respondent made
several demands for payment upon petitioner but these went unheeded.

- petitioner contends that it could not be held liable because the property covered by the
insurance policies were destroyed due to fortuities event or force majeure; that
respondent's right of subrogation has no basis inasmuch as there was no breach of
contract committed by it since the loss was due to fire which it could not prevent or
foresee.
RTC: rendered its decision dismissing respondent's complaint. 

:It held that the fire was purely accidental; That the cause of the fire was not
attributable to the negligence of the petitioner; that it has not been established that
petitioner is the debtor of IMC and LSPI; that since the sales invoices state that "it is
further agreed that merely for purpose of securing the payment of purchase price, the
above-described merchandise remains the property of the vendor until the purchase
price is fully paid", IMC and LSPI retained ownership of the delivered goods and must
bear the loss.

CA:reversed the decision of the RTC.

Issues: WON the insurance Company of North America can claim against Gaisano Cagayan for
the debt that was insured.

 Ruling: In this case, the insurance policies provide coverage for the book debt in connection
with ready-made clothing materials which have been sold or delivered to various customers and
dealers of the insured anywhere in the Philippines; and defined book debts are the “unpaid
account still appearing in the Book of Account of the insured 45 days after the loss covered
under this policy.”

Thus, what were insured against were the accounts of IMC and LSPI with the petitioner which
remained unpaid 45 days after the loss through fire, and not the loss or destruction of the goods
delivered.

Commercial Union Assurance v. Lepanto, GR L-52027, 27 April 1982.

Facts

Issues

Ruling

Harding v Commercial Union, GR L-12707, 10 August 1918

Facts

Mrs. Henry E. Harding (insured), with the consent of his husband, took a fire
insurance policy on her automobile from Commercial Union Assurance Company
(a foreign corporation; insurer), through its local agent, Smith, Bell & Company;
the latter filled out the application form which the insured signed. The automobile
was later destroyed by fire but the insurer denied the claim on the ground that the
application contained false statements and representations. Among others, the
insurer claims that it was deceived as regards the value of the automobile and its
ownership; it argued that the truthfulness of statements regarding the same was
warranted by the insured under its application.

Issues

Should Commercial Union Assurance Co. be held liable?

HELD – YES.

The Supreme Court held the insurer liable, finding no fraud on the part of the
insured. On the issue of the automobile’s value, the Court held that the valuation in
the policy is conclusive as between the parties in the absence of fraud; further, that
the insured did not warrant the same, noting that it is the insurer’s agent who made
out the application. On the issue of ownership, the Court held that the insurer is not
in the position to question the validity of the transfer or donation of the automobile
from the insured’s husband to her and that in any case, it had the burden of proof to
show its invalidity.

Lampano v Jose, GR L-9401, 30 March 1915


Facts: The defendant, Mariano Barretto, constructed a house for the other defendant, Placida
Jose. Subsequently, Jose sold the house to the plaintiff, Antonina Lampano. The house was
destroyed by fire. At the time of the fire Lampano still owed Jose and Jose still owed Barretto on
the cost of the construction. After the completion of the house and sometime before it was
destroyed, Barretto took out an insurance policy upon it in his own name, with the consent of
Jose. After its destruction, he collected P3,600 from the insurance company.
The plaintiff alleged in her complaint that there was a verbal agreement between her and Jose, at
the time of the purchase and sale of the house, to the effect that the latter agreed to deliver to her
the insurance policy on the building; that she did not learn that the policy was in the name of
Barretto until after the fire; and that neither Jose nor Barretto has any right to the insurance or to
the money received therefrom. She prayed for judgment against each of them for the sum of
P3,600, the amount of the insurance collected.
Issue: WON Barreto as building contractor has an insurable interest in the completed building
pending the payment of the construction price.
Ruling:
 It is well settled that a policy of insurance is a distinct independent contract between the insured
and insurers, and third persons have no right either in a court of equity, or in a court of law, to the
proceeds of it, unless there be some contract or trust, expressed or implied, between the insured
and third persons. The trial court did not find that such verbal agreement existed as contended by
the plaintiff.
In the case at bar Barretto assumed the responsibility for the insurance. The premiums were paid
by him without any agreement or right to recoup the amount paid therefore should no loss result
to the property. It would not, therefore, be in accordance with law and his contractual obligations
to compel him to account for the insurance money, or any part thereof, to the plaintiff, who
assumed no risk whatsoever.

There can be no question that Barretto had an insurable interest in the house. He constructed the building,
furnishing all the materials and supplies, and insured it after it had been completed.

Ong Lim Sing, Jr. v FEB Leasing, GR 168115, 8 June 2007

Facts

 On March 9, 1995, FEB Leasing and Finance Corporation (FEB) entered into a lease of equipment
and motor vehicles with JVL Food Products (JVL). On the same date, Vicente Ong Lim Sing, Jr.
(Lim) executed an Individual Guaranty Agreement with FEB to guarantee the prompt and faithful
performance of the terms and conditions of the aforesaid lease agreement. Corresponding Lease
Schedules with Delivery and Acceptance Certificates over the equipment and motor vehicles formed
part of the agreement. Under the contract, JVL was obliged to pay FEB an aggregate gross monthly
rental of One Hundred Seventy Thousand Four Hundred Ninety-Four Pesos (P170,494.00).

JVL defaulted in the payment of the monthly rentals. As of July 31, 2000, the amount in arrears,
including penalty charges and insurance premiums, amounted to Three Million Four Hundred
Fourteen Thousand Four Hundred Sixty-Eight and 75/100 Pesos (P3,414,468.75). On August 23,
2000, FEB sent a letter to JVL demanding payment of the said amount. However, JVL failed to
pay.

CASE HISTORY:

On December 6, 2000, FEB filed a Complaint with the Regional Trial Court of Manila for sum of
money, damages, and replevin against JVL, Lim, and John Doe.

In the Amended Answer, JVL and Lim admitted the existence of the lease agreement but asserted
that it is in reality a sale of equipment on installment basis, with FEB acting as the financier. JVL and
Lim claimed that this intention was apparent from the fact that they were made to believe that
when full payment was effected, a Deed of Sale will be executed by FEB as vendor in favor of JVL
and Lim as vendees. FEB purportedly assured them that documenting the transaction as a lease
agreement is just an industry practice and that the proper documentation would be effected as soon
as full payment for every item was made. They also contended that the lease agreement is a
contract of adhesion and should, therefore, be construed against the party who prepared it, i.e.,
FEB.

In upholding JVL and Lim's stance, the trial court stressed the contradictory terms it found in the
lease agreement where the RTC stated: 

It has also been held that the test of insurable interest in property is whether the assured has a
right, title or interest therein that he will be benefited by its preservation and continued
existence or suffer a direct pecuniary loss from its destruction or injury by the peril insured
against. If the defendants were to be regarded as only a lessee, logically the lessor who
asserts ownership will be the one directly benefited or injured and therefore the lessee is not
supposed to be the assured as he has no insurable interest.

There is also an observation from the records that the actual value of each object of the contract
would be the result after computing the monthly rentals by multiplying the said rentals by the
number of months specified when the rentals ought to be paid.

Still another observation is the existence in the records of a Deed of Absolute Sale by and between
the same parties, plaintiff and defendants which was an exhibit of the defendant where the plaintiff
sold to the same defendants one unit 1995 Mitsubishi L-200 STRADA DC PICK UP and in said
Deed, The Court noticed that the same terms as in the alleged lease were used in respect to
warranty, as well as liability in case of loss and other conditions. This action of the plaintiff
unequivocally exhibited their real intention to execute the corresponding Deed after the
defendants have paid in full and as heretofore discussed and for the sake of emphasis the
obscurity in the written contract cannot favor the party who caused the obscurity.

Thus, the contract is considered  a sale on installment; there is no chattel mortgage on the
thing sold, but it appears amongst the Complaint's prayer, that the plaintiff elected to exact
fulfillment of the obligation.

For the vehicles returned, the plaintiff can only recover the unpaid balance of the price because
of the previous payments made by the defendants for the reasonable use of the units, specially so,
as it appears, these returned vehicles were sold at auction and that the plaintiff can apply the
proceeds to the balance. However, with respect to the unreturned units and machineries still in the
possession of the defendants, it is this Court's view and so hold that the defendants are liable
therefore and accordingly are ordered jointly and severally to pay the price thereof to the
plaintiff together with attorney's fee and the costs of suit in the sum of Php25,000.00.

SO ORDERED.

FEB elevated the case before the CA where the latter declared the transaction between the parties
as a financial lease agreement under Republic Act (R.A.) No. 8556 where it reads:

WHEREFORE, the instant appeal is GRANTED and the assailed Decision dated 22 November 2002
rendered by the Regional Trial Court of Manila, Branch 49 in Civil Case No. 00-99451 is
REVERSED and SET ASIDE, and a new judgment is hereby ENTERED ordering appellees JVL
Food Products and Vicente Ong Lim, Jr. to solidarily pay appellant FEB Leasing and Finance
Corporation the amount of Three Million Four Hundred Fourteen Thousand Four Hundred Sixty
Eight Pesos and 75/100 (Php3,414,468.75), with interest at the rate of twelve percent (12%) per
annum starting from the date of judicial demand on 06 December 2000, until full payment thereof.
Costs against appellees.

SO ORDERED. 

Hence, this Petition for Review was submitted before the SC.

Issue

 Does Lim have an insurable interest over the subject personal properties? 

Ruling
 Yes. The petition is DENIED. The Decision of the CA in CA-G.R. CV No. 77498 dated March 15,
2005 and Resolution dated May 23, 2005 are AFFIRMED. Costs against petitioner.

Petitioner, as a lessee, has an insurable interest in the equipment and motor vehicles leased.
Section 17 of the Insurance Code provides that the measure of an insurable interest in property
is the extent to which the insured might be damnified by loss or injury thereof. It cannot be
denied that JVL will be directly damnified in case of loss, damage, or destruction of any of the
properties leased. 

In view of the foregoing, the stipulation in Section 14 of the lease contract, that the equipment shall
be insured at the cost and expense of the lessee against loss, damage, or destruction from
fire, theft, accident, or other insurable risk for the full term of the lease, is a binding and valid
stipulation

However, FEB is not the right party to file against in this case. The stipulation in Section 9.1 of the
lease contract that the lessor does not warrant the merchantability of the equipment is a valid
stipulation. Section 9.1 of the lease contract is stated as:

9.1 IT IS UNDERSTOOD BETWEEN THE PARTIES THAT THE LESSOR IS NOT THE
MANUFACTURER OR SUPPLIER OF THE EQUIPMENT NOR THE AGENT OF THE
MANUFACTURER OR SUPPLIER THEREOF. THE LESSEE HEREBY ACKNOWLEDGES THAT
IT HAS SELECTED THE EQUIPMENT AND THE SUPPLIER THEREOF AND THAT THERE ARE
NO WARRANTIES, CONDITIONS, TERMS, REPRESENTATION OR INDUCEMENTS, EXPRESS
OR IMPLIED, STATUTORY OR OTHERWISE, MADE BY OR ON BEHALF OF THE LESSOR AS
TO ANY FEATURE OR ASPECT OF THE EQUIPMENT OR ANY PART THEREOF, OR AS TO ITS
FITNESS, SUITABILITY, CAPACITY, CONDITION OR MERCHANTABILITY, NOR AS TO
WHETHER THE EQUIPMENT WILL MEET THE REQUIREMENTS OF ANY LAW, RULE,
SPECIFICATIONS OR CONTRACT WHICH PROVIDE FOR SPECIFIC MACHINERY OR
APPARATUS OR SPECIAL METHODS.

In the financial lease agreement, FEB did not assume responsibility as to the quality,
merchantability, or capacity of the equipment. This stipulation provides that, in case of defect
of any kind that will be found by the lessee in any of the equipment, recourse should be
made to the manufacturer. "The financial lessor, being a financing company, i.e., an extender
of credit rather than an ordinary equipment rental company, does not extend a warranty of
the fitness of the equipment for any particular use. Thus, the financial lessee was precisely in a
position to enforce such warranty directly against the supplier of the equipment and not against the
financial lessor. 

TRADERS INSURANCE vs. GOLANGCO, GR L-6442, 21 SEPTEMBER 1954 -


ALMOCERA

FACTS:
Tomas Lianco and the Archbishop (no name indicated) entered into a contract of
lease on a parcel of land owned by church. As lessee, Lianco erected a building
on the leased portion of the church’s land. Lianco later transferred ownership of
this building to Kaw Eng Si, who later transferred the same to Golangco. This
transfers by Lianco of his right to lease and the building’ownership were without
consent of the Archbishop. The Archbishop filed an ejectment case against
Lianco, who appears to be occupants of the premises building with others paying
rent to Golangco. This right of Golangco to receive rent on the building was
judicially recognized in a case decided between Lianco and some others
occupying the premises pursuant to a compromise agreement. At the moment,
the Archbishop did not exercise his option to question Golangco’s rights as
lessee, as the transfer by Lianco was without the Archbishop’s consent. On April
7, 1949, Golangco applied for fire insurance with Traders Insurance and Surety
Co. of which Golangco was issued fire insurance policy stating “that all
insurance covered under said policy, includes the 'rent or other subject matter of
insurance in respect of or in connection with any building or any property
contained in any building'. On June 5, 1949, fire ravaged the building premises
pursuant of which Golangco requested Trader’s Insurance to pay the insurance
amount of 10,000 including the amount of rent P1,100 monthly. Traders insurance
refused to pay the insurance as pertaining to the rent averring that Golangco has
no insurable interest therein.

ISSUE:
Whether or not Golangco has insurable interest (in the property) on the rent of
the building premises which may lawfully/validly be subject of insurance?

RULING:
YES, Sec. 13 of the Insurance Code provides that “Every interest in the property,
whether real or personal, or any relation thereto, or liability in respect thereof of
such nature that a contemplated peril might directly damnify the insured, is an
insurable interest.”

By virtue of the contract between Tomas B. Lianco and the Archbishop, Lianco
erected the building of which the premises in question form part and became
owner thereof. He transferred the ownership of the premises in question to Kaw
Eng Si, who in turn transferred it to plaintiff Juan Golangco. Lianco and the actual
occupant of the premises acknowledged plaintiff's right to collect rentals thereon
in a compromise agreement which was incorporated in a judicial judgment. Both
at the time of the issuance of the policy and at the time of the fire, plaintiff
Golangco was in legal possession of the premises, collecting rentals from its
occupant. It seems plain that if the premises were destroyed - as they were - by
fire, Golangco would be, as he was, directly damnified thereby; and hence he had
an insurable interest therein (Section 13, Insurance Law).

It is to be noted that the policy so worded indicates that the fire insurance policy
includes 'rent or other subject matter of insurance in respect of or in connection
with any building or any property contained in any building'. The argument of
Trader’s Insurance that a policy of insurance must specify the interest of the
insured in the property insured, if he is not the absolute owner thereof, is not
meritorious because it was the Trader’s, not Golangco, who prepared that policy,
and it cannot take advantage of its own acts to plaintiff's detriment; and, in any
case, this provision was substantially complied with by Golangco when he made
a full and clear statement of his interests to Trader’s manager.

The contract between Lianco and the Archbishop only forbade Lianco from
transferring 'his rights as LESSEE but the contracts Lianco made in favor of Kaw
Eng Si and plaintiff Golangco did not transfer such rights; and hence no written
consent thereto was necessary. At worst, the contract would be voidable, but not
a void contract, at the option of the Archbishop; but this would not deprive
Golangco of his insurable interest until such option were exercised; and it does
not appear that it was ever exercised.

The ejectment case filed by the Archbishop against Lianco did not remove nor
destroy plaintiff’s insurable interest: first, because plaintiff was not a party
thereto and cannot be bound thereby; and second, because the judgment of the
Municipal Court, at least as late as February 14, 1950, had not been executed so
far as possession of the premises were concerned; so that, as far as plaintiff
Golangco was concerned, his right to the premises and to the rentals thereon
continued to exist on June 5, 1949 when the fire took place."

Lopez v del Rosario, GR L-19189, 27 November 1922

Facts:
Benita Quiogue de V. del Rosario, whom we will hereafter call Mrs. Del Rosario, was
the owner of a bonded warehouse situated in the City of Manila.
She was engaged in the business of a warehouse keeper, and stored copra and
other merchandise in the said building.
Among the persons who had copra deposited in the Del Rosario warehouse was
Froilan Lopez.
Del Rosario issued warehouse receipts in the name of Lopez and a certain Zamora.
One of the conditions of the warehouse receipt provides that the goods will be insured.
Del Rosario secured insurance on the warehouse and its contents. The warehouse and
its contents were destroyed by fire. The warehouse was a total loss. Lopez is now
claiming for the value of his copras deposited in Del Rosario’s warehouse. Del Rosario
failed to settle the amount claimed by Lopez.

 Issues:
WON Del Rosario acted as agent of Lopez in taking out insurance on the contents of
the warehouse and is liable for the amount claimed by Lopez.
 

Ruling:
Yes
Del Rosario acted as agent and is liable for the amount claimed by Lopez.
The law is that a policy effected by bailee and covering by its terms his own property
and property held in trust; insures, in the event of loss, equally and proportionately to
the benefit of all the owners of the property insured.
Even if one secured insurance covering his own goods and goods stored with him, and
even if the owner of the stored goods did not request or know of the insurance, and did
not ratify it before the payment of the loss yet it has been held by a reputable court that
the warehouseman is liable to the owner of such stored goods for his share.

 
San Miguel v Law Union Rock Insurance, GR L-14300, 19 Jan.1920
FACTS:
This action was begun on October 8, 1917, in the Court of First Instance of the city of Manila by the
plaintiff, the San Miguel Brewery, for the purpose of recovering upon two policies of insurance
underwritten respectively by Law Union and Rock Insurance Company (Ltd.), and the "Filipinas"
Compania de Seguros, for the sum of P7,500 each, insuring certain property which has been
destroyed by fire. The plaintiff, the San Miguel Brewery, is named as the party assured in the two
policies referred to, but it is alleged in the complaint that said company was in reality interested in
the property which was the subject of insurance in the character of a mortgage creditor only, and
that the owner of said property upon the date the policies were issued was one D. P. Dunn who was
later succeeded as owner by one Henry Harding. Accordingly said Harding was made a defendant,
as a person interested in the subject of the litigation.
The prayer of the complaint is that judgment be entered in favor of the plaintiff against the two
companies named for the sum of P15,000, with interest and costs, and further that upon satisfaction
of the balance of P4,505.30 due to the plaintiff upon the mortgage debt, and upon the cancellation of
the mortgage, the plaintiff be absolved from liability to the defendants or any of them. The peculiar
form of the latter part of the prayer is evidently due to the design of the plaintiff to lay a foundation for
Harding to recover the difference between the plaintiff's credit and the amount for which the property
was insured. Accordingly, as was to be expected, Harding answered, admitting the material
allegations of the complaint and claiming for himself the right to recover the difference between the
plaintiff's mortgage credit and the face value of the policies. The two insurance companies also
answered, admitting in effect their liability to the San Miguel Brewery to the extent of its mortgage
credit, but denying liability to Harding on the ground that under the contracts of insurance the liability
of the insurance companies was limited to the insurable interest of the plaintiff therein. Soon after the
action was begun the insurance companies effected a settlement with the San Miguel Brewery by
paying the full amount of the credit claimed by it, with the result that the litigation as between the
original plaintiff and the two insurance companies came to an end, leaving the action to be
prosecuted to final judgement by the defendant Harding with respect to the balance claimed to be
due to him upon the policies.
Upon hearing the evidence the trial judge came to the conclusion that Harding had no right of action
whatever against the companies and absolved them from liability without special finding as to costs.
From this decision the said Henry Harding has appealed.
The two insurance companies who are named as defendants do not dispute their liability to the San
Miguel Brewery, to the extent already stated, and the only question here under discussion is that of
the liability of the insurance companies to Harding. It is therefore necessary to take account of such
facts only as bear upon this aspect of the case.
In this connection it appears that on January 12, 1916, D. P. Dunn, then the owner of the property to
which the insurance relates, mortgaged the same to the San Miguel Brewery to secure a debt of
P10,000. In the contract of mortgage Dunn agreed to keep the property insured at his expense to the
full amount of its value in companies to be selected by the Brewery Company and authorized the
latter in case of loss to receive the proceeds of the insurance and to retain such part as might be
necessary to cover the mortgage debt. At the same time, in order more conveniently to accomplish
the end in view, Dunn authorized and requested the Brewery Company to effect said insurance
itself. Accordingly on the same date Antonio Brias, general manager of the Brewery, made a verbal
application to the Law Union and Rock Insurance Company for insurance to the extent of P15,000
upon said property. In reply to a question of the company's agent as to whether the Brewery was the
owner of the property, he stated that the company was interested only as a mortgagee. No
information was asked as to who was the owner of the property, and no information upon this point
was given.
It seems that the insurance company to whom this application was directed did not want to carry
more than one-half the risk. It therefore issued its own policy for P7,500 and procured a policy in a
like amount to be issued by the "Filipinas" Compania de Seguros. Both policies were issued in the
name of the San Miguel Brewery as the assured, and contained no reference to any other interest in
the property. Both policies contain the usual clause requiring assignments to be approved and noted
on the policy. The premiums were paid by the Brewery and charged to Dunn. A year later the
policies were renewed, without change, the renewal premiums being paid by the Brewery,
supposedly for the account of the owner. In the month of March of the year 1917 Dunn sold the
insured property to the defendant Henry Harding, but not assignment of the insurance, or of the
insurance policies, was at any time made to him.

ISSUE:

Whether or not the insurance companies are liable to Harding for the balance of the proceeds of the 2
policies.

RULING:
No.  Under the Insurance Act, the measure of insurable interest in the property is the extent to which the
insured might be daminified by the loss or injury thereof.  Also it is provided in the IA that the insurance
shall be applied exclusively to the proper interest of the person in whose name it is made.  Undoubtedly,
SMB as the mortgagee of the property, had an insurable interest therein; but it could NOT, an any event,
recover upon the two policies an amount in excess of its mortgage credit.
By virtue of the Insurance Act, neither Dunn nor Harding could have recovered from the two policies. 
With respect to Harding, when he acquired the property, no change or assignment of the policies had
been undertaken.  The policies might have been worded differently so as to protect the owner, but this
was not done.
If the wording had been: “Payable to SMB, mortgagee, as its interests may appear, remainder to
whomsoever, during the continuance of the risk, may become owner of the interest insured”, it would
have proved an intention to insure the entire interest in the property, NOT merely SMB’s and would have
shown to whom the money, in case of loss, should be paid.  Unfortunately, this was not what was stated
in the policies.
If during the negotiation for the policies, the parties had agreed that even the owner’s interest would be
covered by the policies, and the policies had inadvertently been written in the form in which they were
eventually issued, the lower court would have been able to order that the contract be reformed to give
effect to them in the sense that the parties intended to be bound.  However, there is no clear and
satisfactory proof that the policies failed to reflect the real agreement between the parties that would
justify the reformation of these two contracts.

Spouses Cha v Court of Appeals, GR 124520, 18 August 1997

Facts 
Petitioner-spouses Nilo Cha and Stella Uy-Cha, as lessees, entered into a lease contract with private
respondent CKS Development Corporation (hereinafter CKS), as lessor. One of the stipulations of
the said lease contract states:

18. x x x. The LESSEE shall not insure against fire the chattels, merchandise, textiles, goods
and effects placed in the leased premises without first obtaining the written consent and
approval of the LESSOR. If the LESSEE obtain(s) the insurance without the consent of the
LESSOR then the policy is deemed assigned and transferred to the LESSOR for its own
benefit; x x x

Notwithstanding the stipulation in the lease contract, the spouses insured against loss by fire their
merchandise inside the leased premises for Five Hundred Thousand (P500,000.00) with the United
Insurancewithout the written consent of private respondents CKS.

On the day that the lease contract was to expire, fire broke out inside the leased premises. When CKS
learned of the insurance earlier procured by the spouses (without its consent), it wrote the insurer
(United) a demand letter asking that the proceeds of the insurance contract (between the Cha spouses
and United) be paid directly to CKS, based on its lease contract with spouses.

United Insurance refused to pay CKS. Hence, the latter filed a complaint against the Cha spouses and
United. RTC ordered defendant United to pay CKS the amount of P335,063.11 and defendant Cha
spouses to pay P50,000.00 as exemplary damages, P20,000.00 as attorneys fees and costs of suit. The
CA affirmed the trial court decision, deleting however the awards for exemplary damages and
attorney’s fees.

Issues
Whether or not paragraph 18 of the lease contract entered into between CKS and the Cha spouses is
valid insofar as it provides that any fire insurance policy obtained by the lessee (Cha spouses) over
their merchandise inside the leased premises is deemed assigned or transferred to the lessor (CKS) if
said policy is obtained without the prior written of the latter

Ruling
NO. CKS has no insurable interest in the goods and merchandise inside the leased
premises under the provisions of Section 17 of the Insurance Code. The insurable interest
over said merchandise remains with the insured, the Cha spouses. The automatic
assignment of the policy to CKS under the provision of the lease contract is void for being
contrary to law and/or public policy.

Sec. 18 of the Insurance Code provides:

Sec. 18. No contract or policy of insurance on property shall be enforceable except


for the benefit of some person having an insurable interest in the property insured.

A non-life insurance policy such as the fire insurance policy taken by petitioner-spouses
over their merchandise is primarily a contract of indemnity. Insurable interest in the property
insured must exist at the time the insurance takes effect and at the time the loss occurs.
The basis of such requirement of insurable interest in property insured is based on   public
policy: to prevent a person from taking out an insurance policy on property upon which he
has no insurable interest and collecting the proceeds of said policy in case of loss of the
property. In such a case, the contract of insurance is a mere wager which is void under
Section 25 of the Insurance Code, which provides:

SECTION 25. Every stipulation in a policy of Insurance for the payment of loss,
whether the person insured has or has not any interest in the property insured, or
that the policy shall be received as proof of such interest, and every policy executed
by way of gaming or wagering, is void.

In the present case, CKS has no insurable interest in the goods and merchandise inside the
leased premises under the provisions of Section 17 of the Insurance Code which provides:

Section 17. The measure of an insurable interest in property is the extent to which
the insured might be damnified by loss of injury thereof."

Therefore, respondent CKS cannot, under the Insurance Code a special law be validly a
beneficiary of the fire insurance policy taken by the petitioner-spouses over their
merchandise. This insurable interest over said merchandise remains with the insured, the
Cha spouses. The automatic assignment of the policy to CKS under the provision of the
lease contract previously quoted is void for being contrary to law and/or public policy. The
proceeds of the fire insurance policy thus rightfully belong to the spouses Nilo Cha and
Stella Uy-Cha (herein co-petitioners). The insurer (United) cannot be compelled to pay the
proceeds of the fire insurance policy to a person (CKS) who has no insurable interest in the
property insured.

The liability of the Cha spouses to CKS for violating their lease contract in that Cha spouses
obtained a fire insurance policy over their own merchandise, without the consent of CKS, is
a separate and distinct issue which we do not resolve in this case.

Tai Tong Chua Che v Insurance Com, GR L-55397, 29 February 1988

 FACTS:
Azucena Palomo bought a parcel of land and building from Rolando Gonzales and
assumed a mortgage of the building in favor of S.S.S. which was insured with
S.S.S. Accredited Group of Insurers. 

 April 19, 1975: Azucena Palomo obtained a loan from Tai Tong Chuache
Inc. in the amount of P100,000 and to secure it, the land and building was
mortgaged
 June 11, 1975: Pedro Palomo secured a Fire Insurance Policy covering the
building for P50,000 with Zenith Insurance Corporation
 July 16, 1975: another Fire Insurance policy was procured from  Philippine
British Assurance Company, covering the same building for P50,000 and the
contents thereof for P70,000

Before the occurrence of the peril insured against the Palomos had already paid
their credit due. 

 July 31, 1975: building and the contents were totally razed by fire

Palomo was able to claim P41,546.79 from Philippine British Assurance Co.,
P11,877.14 from Zenith Insurance Corporation and P5,936.57 from S.S.S. Group
of Accredited Insurers but Travellers Multi-Indemnity refused so it demanded the
balance from the other three but they refused so they filed against them. 

Insurance Commission, CFI: absolved Travellers on the basis that Arsenio Cua
was claiming and NOT Tai Tong Chuache. 

 Palomo Appealed

Travellers reasoned that the policy is endorsed to Arsenio Chua,  mortgage


creditor. 

Tai Tong Chuache & Co. filed a complaint in intervention claiming the proceeds of
the fire Insurance Policy issued by travellers. 

 affirmative defense of lack of insurable interest that before the occurrence of


the peril insured against the Palomos had already paid their credit due the
petitioner

ISSUE: 

Whether or not Tai Tong Chuache & Co. has insurable interest
RULING:

 YES. Travellers Multi-Indemnity Corporation to pay Tai Tong Chuache & Co.

When the creditor is in possession of the document of credit, he need not prove
non-payment for it is presumed. 

The validity of the insurance policy taken by petitioner was not assailed by private
respondent. Moreover, petitioner's claim that the loan extended to the Palomos has
not yet been paid was corroborated by Azucena Palomo who testified that they are
still indebted to herein petitioner. 

Chua being a partner of petitioner Tai Tong Chuache & Company is an agent of
the partnership. Being an agent, it is understood that he acted for and in behalf of
the firm. 

Upon its failure to prove the allegation of lack of insurable interest on the part of
the petitioner, Travellers must be held liable. 

Bachrach v British American Insurance, L-5715, 20 December 1910

Facts

 A Fire Policy was created between Bachrach and British American Assurance Company where it
states:

[Fire policy No. 3007499.]

This policy of insurance witnesseth, that E. M. Bachrach, esq., Manila (hereinafter called the
insured), having paid to the undersigned, as authorized agent of the British American
Assurance Company (hereinafter called the company), the sum of two thousand pesos
Philippine currency, for insuring against loss or damage by fire, as hereinafter mentioned,
the property hereinafter described, in the sum of several sums following, viz:
Ten thousand pesos Philippine currency, on goods, belonging to a general furniture
store, such as iron and brass bedsteads, toilet tables, chairs, ice boxes, bureaus,
washstands, mirrors, and sea-grass furniture (in accordance with warranty "D" of the
tariff attached hereto) the property of the assured, in trust, on commission or for which he
is responsible, whilst stored in the ground floor and first story of house and dwelling No. 16
Calle Martinez, district 3, block 70, Manila, built, ground floor of stone and or brick, first story
of hard wood and roofed with galvanized iron — bounded in the front by the said calle, on
one side by Calle David and on the other two sides by buildings of similar construction and
occupation.
Co-insurance allowed, particulars of which to be declared in the event of loss or claim.
The company hereby agrees with the insured (but subject to the conditions on the back
hereof, which are to be taken as a part of this policy) that if the property above described,
or any part thereof, shall be destroyed or damaged by fire, at any time between the
21st day of February, 1908, and 4 o'clock in the afternoon of the 21st day of February,
1909, or (in case of the renewal of this policy) at any time afterwards, so long as, and during
the period in respect of which the insured shall have paid to the company, and they shall
have accepted, the sum required for the renewal of this policy, the company will, out of their
capital stock, and funds, pay or make good to the insured the value of the property so
destroyed, or the amount of such damage thereto, to any amount not exceeding, in respect
of each or any of the several matters above specified, the sum set opposite thereto,
respectively, and not exceeding in the whole the sum of ten thousand pesos, and also not
exceeding, in any case, the amount of the insurable interest therein of the insured at the time
of the happening of such fire.
In witness whereof, the British American Assurance Company has accused these presents
to be signed this 21st day of February, in the year of our Lord 1908.
For the company.

W. F. STEVENSON & Co. LTD.,

"By...............................................,
"Manager Agents."
And indorsed on the back the following:

The within policy and includes a "Calalac" automobile to the extent of (P1,250)
twelve hundred and fifty pesos Philippine currency.
Memo: Permission is hereby granted for the use of gasoline not to exceed 10 gallons
for the above automobile, but only whilst contained in the reservoir of the car. It is
further warranted that the car be neither filled nor emptied in the within-described
building or this policy be null and void.

Manila, 27th February, 1908.

"W. F. STEVENSON & Co. LTD.,

"By.......................................................,
"Manager Agents."

The defendant answered the complaint that they were released from all obligations whatever
under said policy.
They further narrated that Bachrach maintained a paint and varnish shop in the said building
where the goods which were insured were store. Also, he transferred his interest in and to
the property covered by the policy to H. W. Peabody & Co. to secure certain indebtedness due
and owing to said company, and also that the plaintiff had transferred his interest in certain of
the goods covered by the said policy to one Macke, to secure certain obligations assumed by
the said Macke for and on behalf of the insured. That the sanction of the said defendant had not
been obtained by the plaintiff, as required by the said policy.
Moreover, On April 18, 1908, and immediately preceding the outbreak of the alleged fire,
Bachrach willfully placed a gasoline can containing 10 gallons of gasoline in the upper story
of said building in close proximity to a portion of said goods, wares, and merchandise, which
can was so placed by the plaintiff as to permit the gasoline to run on the floor of said second
story, and after so placing said gasoline, he, the plaintiff, placed in close proximity to said
escaping gasoline a lighted lamp containing alcohol, thereby greatly increasing the risk of fire.
Lastly, they maintained that Bachrach made no proof of the loss within the time required by
condition five of said policy, nor did the insured file a statement with he municipal or any other
judge or court of the goods alleged to have been in said building at the time of the alleged
fire, nor of the goods saved, nor the loss suffered.
In reply, Bachrach alleged that he had been acquitted in a criminal action against him, after a
trial duly and regularly had, upon a charge of arson, based upon the same alleged facts set out in
the answer of the defendant. Also on the 20th of April, 1908, given the defendant due notice in
writing of said loss, the defendant, on the 21st of April, 1908, and thereafter on other occasions, had
waived all right to require proof of said loss by denying all liability under the policy and by declaring
said policy to be null and void.
After hearing the evidence adduced during the trial of the cause, the lower court found that the
defendant was liable to the plaintiff and rendered a judgment against the defendant for the
sum of P9,841.50, with interest for a period of one year at 6 per cent, making a total of P10,431.99,
with costs.

Issues

 Is the annulment of the insurance policy valid on the ground that it was entered into a chattel
mortgage?

Is the insurance policy being in force at the time of said fire, and that the acts or omissions on the
part of the insured which cause, or tended to cause, the forfeiture of the policy, were waived by the
defendant?

Ruling

 No. For all the foregoing reasons, we are of the opinion that the judgment of the lower court should
be affirmed, and it is hereby ordered that judgment be entered against the defendant and in favor of
the plaintiff for the sum of P9,841.50, with interest at the rate of 6 per cent from the 13th of July,
1908, with costs. So ordered.

Upon reading the policy of insurance issued by the defendant to the plaintiff, it will be noted that
there is no provision in said policy prohibiting the plaintiff from placing a mortgage upon the property
insured, but, admitting that such a provision was intended, the lower court has completely answered
this contention of the defendant. He said, in passing upon this question as it was presented:

It is claimed that the execution of a chattel mortgage on the insured property violated what is
known as the "alienation clause," which is now found in most policies, and which is
expressed in the policies involved in cases 6496 and 6497 by a purchase imposing forfeiture
if the interest in the property pass from the insured. (Cases 6496 and 6497, in which are
involved other action against other insurance companies for the same loss as in the present
action.)
This clause has been the subject of a vast number of judicial decisions (13 Am. & Eng.
Encyc. of Law, 2d ed., pp. 239 et seq.), and it is held by the great weight of authority that the
interest in property insured does not pass by the mere execution of a chattel mortgage and
that while a chattel mortgage is a conditional sale, there is no alienation within the meaning
of the insurance law until the mortgage acquires a right to take possession by default under
the terms of the mortgage. No such right is claimed to have accrued in the case at bar, and
the alienation clause is therefore inapplicable.
As to holding that the policy of insurance was in force at the time of said fire and that the acts or
omissions on the part of the insured which caused or tended to cause a forfeiture of the policy were
waived by the defendant, the lower court, in discussing this question, said:

Regardless of the question whether the plaintiff's letter of April 20 (Exhibit B) was a sufficient
compliance with the requirement that he furnish notice of loss, the fact remains that on the
following day the insurers replied by a letter (Exhibit C) declaring that the "policies were null
and void," and in effect denying liability. It is well settled by a preponderance of authorities
that such a denial is a waiver of notice of loss, because if the "policies are null and void," the
furnishing of such notice would be vain and useless. (13 Am. & Eng. Encyc. of Law, 347,
348, 349.) Besides, "immediate notice" is construed to mean only within a reasonable time.
Much the same may be said as to the objection that the insured failed to furnish to the
insurers his books and papers or to present a detailed statement to the "juez municipal," in
accordance with article 404 of the Code of Commerce. The last-named provision is similar to
one appearing in many American policies requiring a certificate from a magistrate nearest
the loss regarding the circumstance thereof. A denial of liability on other grounds waives this
requirement (O'Niel vs. Buffalo Fire Insurance Company, 3 N. Y., 122; Peoria Marine Ins.
Co. vs. Whitehill, 25 Ill., 382), as well as that relating to the production of books and papers
(Ga. Home Ins. Co. vs. Goode & Co., 95 Va., 751; 66 Jur. Civ., 16). Besides, the insured
might have had difficulty in attempting to comply with this clause, for there is no longer an
official here with the title of "juez municipal."

Besides the foregoing reasons, it may be added that there was no requirement in the policy in
question that such notice be given.

San Miguel vs  Law Union Rock Insurance, GR No. L-14300, 19 January 1920

Facts:

On Jan. 12, 1918, Dunn mortgaged a parcel of land to SMB to


secure a debt.
Mortgage contract stated that Dunn was to have the property
insured at his own expense, authorizing SMB to choose the
insurers and to receive the proceeds thereof and retain so much of
the proceeds as would cover the mortgage debt.
Dunn likewise authorized SMB to take out the insurance policy for
him.
Brias, SMB’s general manager, approached Law Union for
insurance to the extent of 15T upon the property.  In the
application, Brias stated that SMB’s interest in the property was
merely that of a mortgagee.
The Law Union, not wanting to issue a policy for the entire amount,
issued one for P7,500 and procured another policy of equal amount
from Filipinas Cia de Seguros.  Both policies were issued in the
name of SMB only and contained no reference to any other interests
in the property. Both policies required assignments to be approved
and noted on the policy.
Premiums were paid by SMB and charged to Dunn. A year later, the
policies were renewed.
In 1917, Dunn sold the property to Harding, but no assignment of
the policies was made to the latter.
Property was destroyed by fire.  SMB filed an action in court to
recover the policies.  Harding was made a defendant because by
virtue of the sale, he became the owner of the property, although
the policies were issued in SMB’s name.
SMB sought to recover the proceeds to the extent of its mortgage
credit with the balance to go to Harding.
Insurance Companies contended that they were not liable to
Harding because their liability under the policies was limited to the
insurable interests of SMB only.
SMB eventually reached a settlement with the insurance companies
and was paid the balance of it’s mortgage credit.  Harding was left
to fend for himself.  Trial court ruled against Harding.  Hence the
appeal.

Issues:

W/N San Miguel has insurable interest as mortgagor only to the


extent of the mortgage credit

Ruling:
YES. 
Section 19 of the Insurance Act:
a change of interest in any part of a thing insured unaccompanied
by a corresponding change of interest in the insurance, suspends
the insurance to an equivalent extent, until the interest in the thing
and the interest in the insurance are vested in the same person.
Section 55: the mere transfer of a thing insured does not transfer
the policy, but suspends it until the same person becomes the
owner of both the policy and the thing insured.
 
With respect to Harding, when he acquired the property, no change
or assignment of the policies had been undertaken.  The policies
might have been worded differently so as to protect the owner, but
this was not done.

You might also like